m c* -SO REESE LIBRARY Of THK UNIVERSITY OF CALIFORNIA. Received. s^^L^Zl^ rSS(/ Accessions No.2*£~&fi*^ Shelf No * £ KEY TO ROBINSON'S UNIVERSITY ALGEBRA; CONTAINING, ALSO, A SHORT TREATISE ON THE INDETERMINATE AND DIOPHANTINE ANALYSIS. AND SOME MISCELLANEOUS EXAMPLES, JBesffltie'O for 5Teacf)ers attfi SatuUents. NEW YORK: IVISON & PHINNEY, 321 BROADWAY CINCINNATI: JACOB ERNSf CHICAGO : 8. C. GRIGGS & CO., 39 & 41 LAKE ST. ST. LOUIS : KEITH k WOODS. BUFFALO : PHINNEY & CO. 185 9. ■ft Entered according to act of Congress, in the year 1847, by HORATIO N. ROBINSON, in the Clerk's Office of District Court of the District of Ohu* \^ V OF THE ' \\ 'UNIVERSITY' flfe A Key, to a mathematical work, is very proper in its place ; but to be constantly at hand, and consulted too often, might prove injurious : we must not, how- ever, confound the improper use of a thing with the thing itself. Those who condemn keys, in general terms, should condemn teachers also ; for a key is neither more nor less than a teacher, in another shape. The self-taught are generally sound and vigorous ; but if they disregard the works and teachings of others, they will be found to be wanting in that certain sym- metry and polish of mind, so characteristic of educated men. So it is with an algebraist ; h'e may go through his text-books, solve every problem, independent of all external aid, and if he does not compare his work with the works of others, he cannot know whether he is skillful or otherwise ; for it is only by comparison that we measure excellence. No solution of a prob- lem, or of an equation, should be called good, if better can be found ; hence it is important that more than one standard of attainment should be before the pupil ; and those who really become eminent, in any science, are those whose talents and dispositions enable them to gather knowledge from every possible source. PREFACE TO REVISED EDITION. In 1857 the author and publisher thought it advisable to enlarge the University Algebra by some twenty-four pages, and some more than twenty additional examples. Solutions of these are inserted subsequent to page 76. Also, solutions of several other problems are inserted, which were omitted in the former editions of this work. 'UNIVERSITY KEY TO ROBINSON'S ALGEBRA SECTION II. CHAPTER I. EQUATIONS. None of the questions in this chapter require the aid of a key, until we come to the 15th, page 65. (15.) (4x—4a \4 I6x—I6a 4a /4x—4a \4_J \ 3 /3~~ = his 9 3 stock at the commencement of the third year, before his ex- penses are taken out. tj /16a? — 16a 4a \4 Hence, ( a }_ = 2a; V 9 3/3 Reduced gives x= 14800, Am. (16.) Put a=99, #=time past. Then a — x== time to come, and per question, 2a? 4a — 4x 3 5 (17.) Let x= the whole composition. Then per question, z=54. KEY TO ALGEBRA. 2a? , r-10=nitre. 3 x 4£ =sulphur. 6 2x 10 — -f— — 2= charcoal. 21 7 2x x 2x 10 By addition, — +-- f h3H =x J 3 6 21 7 Multiply by 6, and drop 5a; from both sides, and we have 4x 60 7 7 or, 4#+21-7+60=7# . • . .#=69. (18.) Puta=183 ; #=what the 1st received ; then a — x=2d received. 4a? 3a — 3a? Per question, — = #=63. 7 10 (19.) Put a=68, #= the greater part, and a — x= the less. 84— #=3(40— a+ x) #=42. (20.) The distance from A to B put =2#. The distance from C to D " =3x. Then, 3 times the distance from B to C must be x Sx , xx — | or the distance is, — | — 2 2 6 2 x x Hence the whole distance is, 5x-\ 1 — =34. 6 2 (21.) Let x= the flock. 2x The first party left him 6. x The second left 3—10=2. 3 EQUATIONS. 7 (23 ) Observe that for every vessel he broke he lost 12 cents : 3 cents fee and 9 cents forfeiture. 300—12^=240 ... - #=5. (24.) Had he not been idle he would have been entitled to ab cents. But he was idle x days at a loss of (b-\-c) cents. Hence, ab — (b-\-c)x=d. ab — d b+c (25.) Put 5x= Then a — 5x = = less part the greater part. Per question, 3 a — 7x = 2 Ox (a — 5x) or, or, Therefore, 7a—49x= 1 40#— 3a+ 1 Sx 204.r=10a=10-204 #=10 5^=50= the less part. (26.) Let 8#=the price of the horse. Then a — Sx= chaise. a=341. 5 Per question, 2« — 16a; — ■3#=24# (a — Sx) 5 or, 2a=4Sx (a— 8x) 14«=301#— 5a+40x 19a=341# or, #=19 8#=152 Arts. (28.) Let 5x= his money. After he first lost and won 4 $., he had 4#-f4 He again lost and won, and then had 3#+3-f-3. £ of this must equal 20, or, 3x-\- 6=24. x=6 5#=30 Ans. KEY TO ALGEBRA. (29.) Let 3x— the income. Then 2a? = the family support. x -=-=70. Hence, .... 3a?=70-9. 3 o 30, 31, 32, and 33 require no explanation. (35.) Last year the rent was x dollars. _ . 8a; This year it is x-\ =1890 100 (36.) is the (35) in general terms. (37.) Let 7a?= the income. Then x= A's annual debt. 7a? — = what B saves. 5 7a? a?=li 5 5 or a?=40. In general terms, 7a?=280, Am, 7a? a ~5 X ~2 5a x= — 4 (38.) xx 3 4 2a? T =a 7a?=£(35d). (39.) Let 10a?= the income. Then 2a?-f-100=the sum spent. 5a?+ 35= « sum left. 7a?-f-135=10a? the whole or 45=a? 450 Am (40.) Let 21a?= the income. Then 3a?-|-«= the sum spent. 7a?-f&= " sum left. 10a?+a-K>=21a?= the whole. («-H) x= . 11 EQUATIONS. (41.) 2#-h4 : 3x-f-4 : (42.) Let x* — 7= the number. Then, per conditions, x — l=Jx* — 7 a*— 2a?+ 1=^—7 or, x=4 and a? 2 — 7=9. (43.) A's rate of travel is J miles per hour. B's rate of travel is f miles per hour. A is in advance when B sets out, 5 / miles. Let a;= the hours after B starts. 5x 7x 56 Then, — = — 1 Reduced gives . • . • . a?=42. 3 5 5 CHAPTER II. EQUATIONS IN TWO UNKNOWN QUANTITIES. (6.) Add the two equations together, representing (x-\-y) by s, and we have 5$-f-3s=50 or 3=5*3. But aH-9y=21-3 Subtract x-\- y= 5*3 8y=16-3 or, y=6. (7.) By adding the two equations we have 55=50 or, x-\-y= 10 but 4#-{-2/=34 Hence, 3a? =24 or, «a;=8. (9) and (10) are resolved same as (6) and (7). (11.) From the first equation we have y=2x~- 80. Transpose —8 in the second equation and we have x-\-v x 2y — x , 9 1 -=— +35. 5 ' 3 4 10 KEY TO ALGEBRA. Multiply by 60 and we have 12aH-122/+20;r=30i/— 15z+35-60 or 47a?= 183/+35-60 Substituting the value of 18y, we have 47#=36;r— 1 8-80+35 -60 or ll;r=— 240-6+350-6=110-6 Hence, x=60. (14.) Bringing unknown terms to the first members of the equation and we have 4 4_ 4 2__3 x y y x~~2 2 1 By addition, -=- or x=4. x 2 (15.) Put a=50. Then, #+3a : y — a : : 3 : 2 And x — a : y-\-2a : : 5 : 9 2£+6a=3y— 3a (1) 9x— 9a=5y+10a (2) Multiply (1) by 5, and (2) by 3 ; then, 10a?+30a=15?/— 15a (3) 27#— 27a= 1 5y + 30a (4) Subtract (3) from (4) and 17a? — 57a=45a 17#=102a ar=6a=6-50=300. (16.) Divide the numerator of the second member of the first equation by its denominator, and we have — lla>— 14y+127y 3*+6 y +l=3*+6 y +l+ to-J+8 Hence, llar + 142/=127 (1) Multiply the second equation by (Sy — 4) and we shall have EQUATIONS. 11 (151 — 16a?) (3y— 4) 9xy—l2x=± '-XZ --f-9ay— 110 (151— 16#)(3y—4) or, 110— 12z=- ili L 4y— 1 440t/— 48#y— 1 10+12#=453y— 48#t/— 604-f-64a? 0=13i/+52;r— 494 or, 4#+y=38 (2) Add (1) and (2), and we have l5(x-\-y)=165 or, a?+y=ll (3) (3) from (2) gives 3#=27 x=9. (17.) Multiply the 1st equation by 14 and we have 42;r— 7y=49 Add — a;+7y=33 41x =82 or, «=2. (18.) 2 aH-g2/=16 5 2 3 Subtract -yA — =19 3* 5 Kty+9y=19-15 y=15. (19.) Divide 2d by the 1st, and x—y~Z But x+y=8. (20.) Multiply the first equation by (x+y), and the sec- ond by 9, and we have 4(z-j-y) 2 =9(z a — y*) 9(z*— y*)=9-36 4(a?-{-y) 2 ==9-36 Hence, x+y=9 Divide the 1st equation by this last, and we have • x — #=4. 12 KEY TO ALGEBRA. (21.) x= Jy 3 a*= 64i/ 3 64y» ~27~ ■^=37 27 372/ 8 =37-27 y=3. (22) and (23) require no remark. (24.) The first equation gives #4-242/= 91 (i) Add 40tf-|- y=763 (2) Multiply (1) by 40, and subtract equation (2), and 959y=2877 or, y=3. (25.) From 1st equation take the 2d, and we have 2hx+5y=60. Divide by 2\ and we have x-\-2y=24 But ix+2y=\9 \x = 5 or, a:=10. (26.) Add the two equations, and 2(*+y)+£(*+y)+ 20 =*+y or, -+-+20=$ 2 3 By 2d equation, 5=120. £(120)— 5=y=35, JSm. CHAPTER III. EQUATIONS OF THREE OR MORE UNKNOWN QUANTITIES. (7.) Given '2x=u-\-y-\-z 3y=M-f-a?-fz 4z=u-)rx-\-y w=a?— 14 Subtract 2d from the 1st, and 2x — Sy=y — x or Subtract 2d from the 3d, and 4z — 3y=y — z or > to find u, x, y and z. 3x=4y (1) 5z=4y (2) EQUATIONS. Add 3d and 4th and 4z=2x-\-y — 14 Multiply (3) by 5 and (2) by 4 and 202=10aH- 5y— 70 202= 16i/ 13 (3) 0=10a?— lly— 70 or 30#— 33i/=210 But, equation (1) 30#— 40iy= 7i/=210 y= 30 ~x y z * 2 3 4 =a+b (8.) 3 4 5 b J>=a — 4 x t y z .4 5 6 =a — b To avoid numeral multiplication, and really, to under- stand algebra as applied here, observe that 62+38=100. Put a=50; then 62=a-f-12=a+6. Clearing of fractions, we have 6aH- 4y+ 3z=12a-f-12& (1) 20#-f- 1 5i/-f- 12^=60a— 156 (2) 15a?+12y-r-102r=60a— 606 (3) Multiply (1) by 4 , and subtract (2). Then, 4#+y=636— 12a (4) Subtract (3) from (2,) and 5x+3y+2z= 456 3 Subtract 15#-f9i/4-6z=1356 12a?-j-8y-f-6z= 246+24a (5) 3x+ y =1116— 24a Subtract (5) from (4), and we have #=(12a— 486)=12(a— 46)=12-2, Ms. 14 KEY TO ALGEBRA. That is, a?=24 or 26. Now, equation (4) gives us 8b-\-y=G3b—ba y=(55— a)6=5-12=60. (9.) By adding the three equations and reducing, we have 4x-\-3y-\-2z=3a (1) By adding the 2d and 3d, reducing and doubling, we have l0x-\-4yi~2z=4a (2) Subtracting (1) from (2), and we have 6x-\-y=a (3) Adding the 1st and 3d, and reducing, we have x-{-2y—a (4) From (3) and (4) we readily find x and y. —2z =40 (1) -\-3z =35 (2) (10.) <> 3u +* =13 (3) y +u -W=15 (4) jx—y-\-3t— w =49 (5) It is easier to eliminate t than any other letter. Subtract (3) from (4), and we have y— 2u=2 (6) Three times (3) taken from (5), gives 3x—y—\0u=\0 (7) Add (6) and (7) and divide the sum by 3, and x — 4w=4 (8) Double (6), and subtract it from (8,) and we have x=2y (9) Eliminate z from equations (1) and (2), and we have 4#-Hlt/=190 But4a?=8#. Then 19t/=190, or y=10. C2x +y 4y — a: < 3 EQUATIONS. 15 PROBLEMS PRODUCING SIMPLE EQUATIONS, INVOLVING TWO OR MORE UNKNOWN QUANTITIES. (1.) Let a?, y, and z represent the num a^=600 a?z=300 1/2=200 Multiply (1) and (2), and divide the product by 3, and we have x 2 =900 ar=30. (2.) Let x, y, and z represent the numbers. Then, per question 70 (2) 5 * +2/+2=l 90 (3) Double (1) and subtract (3), gives #=50. This problem calls the pupil's judgment into exercise. He does not know in the first place which is greatest, x or z ; hence he must try both suppositions, and the one that verifies equation (2) is right. (3.) Let x,y, and z represent the shares, and put a=120 y — $(x-\-z)=a z— §Gr-fy)=a Clearing of fractions, we have Ix—Ay — 4z=7a (1) — 3x+8y—3z=8a (2) —2x— 2y-\-9z=9a (3) Double (1) and to the product add (2,) and we have Ha? — ll*=22a X— z— 2a (4) 16 KEY TO ALGEBRA. Double (3), and to the product add (1), and we have — llH-22z=lla — H- 2z= a (5) Add (4) and (5), and we have z=3a=360. (4.) Resolved in the book. Let x, y, u, and z represent their ages, and s their sum. Then, *— *=18 s—u=ld s — y=14 s — a?=12 "Rxr orl/litirm In fift . ■ . , . • t=20 r>y auuiuun, os — uu • • • («5.) Let #=A's shillings. • • O^^AiXfrn y=B's " z=C's " After the first game they will have as here i •epresented x — y — z=A 2y =B 2z =C After the second game, 2x— 2y— 2z-—k Sy — x — z=B 4z =C After the third game 4a? — 4y — 4z= 16 CD 6y—2x—2z= 16 (2) Iz — x — y= 16 (3) (*) Sum, x-\- y-\- 2r=3'16 Add (3) and (4) and we have 8z=4 16 . . . z=8. (6.) This problem is resolved in the book, by equation 7, Art. 53. EQUATIONS. 17 (7.) Let x represent the better horse, and y the poorer. ar+10=H(y+ ls ) Therefore, J(y+10H|(y+15)+5 Reduced gives i/==50. (8.) Let #= the price of the sherry. y= brandy. Put a=78. 2a?-}- y=3a 7x+2y=9a+9 3a? =3a-f-9 x = a+3=. . 81a (».) Let x=X's time. y=B's time. Then, 1 , -=the part that A can do in one day. X And 1 _=the part that B can do in one day. y 4 4 4 1 x y 16~~4 36 3 — =- Hence y 4 (10.) 2a? 2 x-\-2 3 y+7 3 2y 5 3a?=y-f-7 5a?+10=6y. (11.) , 2 y , 3 * (12.) : Let x= the greater, and (24 — a?)= less. a? 24 — x ..4.1 24 — a? a? a? 2 : (24— a:) 2 : : 4 : 1 By evolution, x : 24 — x : : 2 : 1. ♦ 2 y=48. 18 KEY TO ALGEBRA. (13.) Let a?= the number of persons. yz= what each had to pay. Then, xy= the amount of the bill. Put (x+4)(y— 1)= the bill. Also, (as— 3)(y+l)= the bill. xy + 4i/ — a? — 4 = #?/ 4y — x — 4=0 — 3y-{-x— 3=0 By addition, y — 7=0 (14.) \0x-\-y=4x-\-4y or, y=2x. \0x+y+27=l0y-\-x or, 9x +27= 9y x + 3= y=2x hence, a?=3. (15.) Let rr= the digits in the place of 100's. y= " in the place of 10's. z= «* the units. x-\-y+2=ll z=2x 100.r+10y+2r+297=100z+10i/+a: 99^+297 = 99* x-\-3=z=2x Hence x=3. x— -40 #—20 , #—10 (16.) Let , , and represent 2 3 4 the parts. Then x — 40 x — 20 a?— 10 1 1 =90 #=100. 2 3 4 (17.) Let x represent the part at 5 per cent, and (a — x) the part at 4 per cent. Then bx 4a— 4x . + =6 100 100 Hence x=100&— 4a. EQUATIONS. 10 (18.) To avoid high numerals, and of course a tedious operation, Put a=5000 ; then 2a=10000, 3«=15000, 3a 16a — =1500, and =800. 10 100 Put #= A's capital, and r — 1 = A's rate. x-\-2a— B's " r= B's rate. a?-j-3a= C's " r+l= C's rate. ~rx — x 16a rx-\-2ar By conditions, 100 100 100 rx — x 3a rx-\-3ar-\-x-^3a 100 10 . 100 Reducing (1), gives a?=(16 — 2r)a '27— 3r (1) (2) /27 — 3r\ " (2), " *=(— — y Hence, 32 — 4r=27 — 3r, or r=*=5. (19.) Put a=1000, x and y to represent the tw< parts, and r and t the rates expressed in decimals, Cx+y=13a (i) Then by conditions,^ y ' " ^ ' ' ] te=360 (3) t ry=490 (4) Divide (3) by (4), and we have 36 (5) C-)Q J 49 x t From (2) we have -=- y r Substitute the value of - in equation (5), and T x* 36 6v --= — or x= — y* 49 7 6v By returning to equation (1) we have -— +2/=13e*. •13y=13a*7 or. y=7a. 20 KEY TO ALGEBRA. (20.) Let x, y, and z, represent their respective ages. Then by conditions given, x — y= z 5y+2z— x=U7 x+y+z= 96 (21.) Let x, y, and z, represent the respective property of each, and put s=their sum. rx+3y+3z=47a a=100. Conditions, J y _|_ 4 X +4z=58a Lz+5x-\-5y=63a Add 2x to the 1st equation, 3y to the 2d, and iz to the 3d, observing that x-\-y-\-z=s ; then we shall have 3s=47a-f-2;z (1) 4s=58a+3y (2) 5s=63a-f4z . (3) 3s— 47a or, 2 4s — 58a 5s — 63a z= 4 3s — 47a 4s— 58a 5s— 63a By addition, s= h H \ 2 3 4 Hence, s=19a. This value of s, put in equation (1), gives #=5a=500. (22.) Taken from the book we have Is — p ms — a ns — r s=-—f-+ 1+ / — 1 m — 1 n — 1 or , ,~(±.y.r+(JL Y__i_ +(JL y__ L. V/-1/ /_i \ot-1/ m-1 \nr-l/ n-l (I m n \ p q r — + -f- 1 )s=-i--r— H — -f EQUATIONS. 21 4-1- ?■■!■■.»- / — 1 m — 1 w — 1 Hence, s= =k I m n I — 1 m — 1 n — 1 As s is now known, we call its value k. lk —P Then, #= -- mk — q nk — r r— 1 (23.) Let x, y, and z represent the respective sums. x +r a (i) y+5=« (2) *+i=« (3) 2x-\-y=2a 3y-\-z=3a or, 4z-\-\2y—\2a 4z-\-x=4a or, 4z± x= 4« — x+l2y= 8a From the 1st 24x-\-l2y=24a 25# =16a (24.) This problem is resolved in the work, by the 13th example, page 80, (Art 51.) (25.) Let x= the greater, and y the less. 5#-Hy=13 jx — 5y=0 or, 2x=3y. 22 KEY TO ALGEBRA. (26.) #-K(y+z)=a=51 y+*{x-\-z)=a z-\-k(x+y)=a x+(x-\-y+z)=2a 2y+{x+y+z)=3a 3z-}- (x-\-y-\-z) =4a x—2a — s (1) y =i(3a— s) (2) *=i(4a— 8 ) (3) s= 2a — s-f-£(3a — s)-H(a — s) 6«= 12a— 6s+9a-— Ss-\-Sa—2s I7s=29a or 5=29-3=87. Now equations (1), (2), and (3), will readily give x, y, and z. (27.) Let #=A's, y B's, and z=C's sheep. Then by the conditions, x+8 — 4=y-fz— 8 5(y+8)="ar+ar— 8 *(*+8)=ar+y— 8 *4-12=» y+* (1) y+24=2x+2z (2) z-\-32=3x-\-3y (3) Add (1) and (3), and we have x-\-44—3x-\-4y. Double (1), and subtract (2), and we have 2x — y—2y — 2a? or, 4ar=3y But 2#-f4y=44 4#+8i/=44-2 lly=44-2 or y=8. (28.) ;r+l_l x _l "J 3 y+l~4 (29.) a?+2__5 a: _1 "^ 7 ^"S EQUATIONS. S3 (30.) This is a repetition of the 10th example, page 89, inserted here by oversight. (31.) Let #=A's money, and 2/=B's. *-5=i(y+5) (1) ar-r-5=3y — 15 (2) Subtract (1) from (2), and we have 10=3y— 15— or y=ll. 2 Z (32.) Let a?= the number of bushels of wheat flour. And y= barley " Then the cost of the whole will be expressed by iOx-\-4y The sale at 11 shillings will be llic-f-lly Now by the conditions, 10aH-4y : ll#-f-lty : : 100 : 143| Multiply the last two terms by 4, and \0x-\-4y : llx-\-lly : : 400 : 575 Divide the two last terms by 25, and 10aH-4y : ll^-flly : : 16 : 23 bx+2y : llz-fllt/ : : 8 : 23 115#-}-46y=88;r-f88i/ 27^=42y 9x=-\4y These co-efficients, 9 and 14, give the lowest proportion in whole numbers. The proportion was only required. (33.) Let lOx -jry represent the number. Kito+sO-e-H i(io*-h/)= /708 = \2 : : 8 : cube required. : : 8 : correction. DS IN GENERAL. (2.) : ^/49a 2 X2 = 7aj2 (3.) = JWXZy =2xjSy (4.) :y27£ 3 : K2x =3x*j2x (*•) =4 727. X4 = 12 V4 (6.) Jx? — aV= 3 JS2a 3 = J2$a 3 x 2 = = Jx 2 (x- -a*) —xjx — a* (*•) = V80 3 X4 i =2a V4 (8.) = t j4a 2 x 2 X7a =2aav7a (9.) Performed in the work. V 64 V 64 V 64 4 V (12.) /50 /25-2 /25-6 5 _ PURE EQUATIONS. $7 (i3.) * v^^6" 2 =v^(TT6T=« vh^ (14.) /2a /T 1 — 3 9 3 V (Art. 81.) Requires no aid from a key. (Art. 82.) (I.) 5^5X3/8 = 15^X2X4=30710 (2.) 4712x3^2-12724=24^/6 (3.) 372 X 278=6716=24 (*•) 2 yuX3 3 74=6 3 77X8=12 8 77 (5.) 275 x 27X0=4750=2072 (1.) Here to multiply (a-f-^) 3 by (a-f-6) we must sim- ply add the exponents. (2.) (7)^X7^=7^^=7^ (3.) 2(3)* X3(4)*=2(3)^X3(4)*=2(27)*X 3(16)*== 6(27X16)*=6(432)~« (4.) (15)*X(10)*=(15)*X(10)*=(225)*X(1000)*= (225000)* PURE EQUATIONS. (14.) Multiply both members by Ja-\-x % and t Jax-Jrx 2 -\-a-{-x=i=2a jjax+x^a — x ax-\-x*=a 2 — 2ax J rx % 3ax=a* x*=la. KEY TO ALGEBRA. (15.) Multiply by Jtf+x 2 , then xJa 2 +x 2J r a 2 -\-x 2 =2a i xJa 2 -\-x 2 =a 2 — x 2 By squaring both members and reducing we have Sa 2 x 2 =a 4 or x=±ajl, (16.) Square both members, and x 2 +2ax+a 2 =a 2 +xj¥+x* Drop a 2 and divide by x, and x-\-2a=Jb 2 -\-x 2 Square, &c. (17.) Divide the numerator by its denominator, in each member, and we have 4 15 1, =1 v/6a?-r-2 4vQx-\-Q Drop 1 and change signs, and clear of fractions, and 16^6^-1-24=1576^+30 Hence x=6. (18.) Cube both members, and (4-\-xf 64-f-z 2 — Sx=- -=(4+x )2 (4+*) K T J Hence, 64-\-x 2 —8x=16-\-8x-\-x 2 or x=3, (19.) By clearing of fractions, f>-\-x-\- J x 2 -\-5x=\5 By reduction, 1 Jx 2 -{-5x=lO — x Square &c. A/ x-f-Jx — \/ x — Jx= -—= V x+Jx M iltiply by \f x-\- Jx, and we have X \ w X i^ Ju *L 2 /^ X 2a:+ 2 Jx — 2 Jx 2 —x =3jx 2x — Jx==2 s /x 2 — x 4X 2 — 4x,Jx-{-x=4x 2 — 4a? *=H* PURE EQUATIONS. 29 (21.) Resolved in the work. (22.) Resolved the same as 17. 2b 7b 1 =1 a \/ax-\-b 3Jax*\-5b _ _ 96* Hence, 6Jax+l0b=7jax-{-7b or x- ( U. 136. ) ( & 136. ) (23.) Square both members and we have ^+a^ 2 -f-12= / i+2;r+z* l Jx 2 -\-\2=2-\-x Square, &c. . x=2. (24.) Multiply numerator and denominator by the nu- merator, and we have i Take square root, and transpose ,j4x, and N /17-Fl=3— J4x Square, 4a?+l=9 — 6,j4x-{-4x Hence, #=£. (25.) Square root gives a — x=Jb or, x=a — Jb. (26.) Clearing of fractions and we have 271—^=73 4— 4;z 2 =3 x 2 =% (27.) Take the square root of both members, and 2 1 =- or 4=a? — 1 x—l 2 (28.) Resolved the same as the 21st. (29.) Clearing of fractions we have A /r i ^M-a>-9=36 30 KEY TO ALGEBRA Jx 2 — 9#=45 — x x 2 — 9^=452—90^+^ 81z=45-45 9-9#=5-9-5-9 #=5-5=25. (30.) Resolved the same as (17) and (22.) Dividing numerators by denominators, we have 10 10-5 3 =3 jx+2 V^-Ho Drop 3 from both sides, change signs, and divide by &, and clear of fractions, and 2 > /#-f80=21 > /x+42 Hence x=4. (31.) Multiply numerator and denominator of the first member by (Jx+Jx — a) Then, y+=x*. 8 V 3 — 1/ 3 =56 y=8 y=2. 32 KEY TO ALGEBRA. (11) (12) and (13) resolved in the work. U4.) or 2 — v 8 -=6 or a?+v=6 x—y * From the second, #y=5. (15.) Divide the 1st equation by (x-\-y), and x 2 — xy-\-y t —2xy x 2 — 2xy-\-y 2 =xy=lQ (1) Add 4xy =64 a?-\-2xy+y*= 80=16-5 Square root, x-{-y=4 ) j5 Square root of (1) x — y=4 2x =4^5+4 (19.) Double the 2d equation, and add and subtract it from the 1st, then x 2 -\-2xy-\-y 2 =a-\-2b x 2 — 2xy-{-y 2 —a — 26 z+y=Ja+2b x—y=Ja — 2b (Art. 92.) (5.) Add the two equations and extract square root, and i i we have x* -\-y* =±4 (1) Separate the first member of the first equation into fac- tors, and we have # 5 Cz 5 +i/ 5 )==12 (2) Divide (2) by (1) and a?*=d=3 x=9. (6.) is of the same form and resolved the same as (5.) (?.) Add the two equations, and extract square root, and 3 3 we have x* -\-y* = J a-\-b \ PURE EQUATIONS. 33 But a?* Gr* +#*)=« I a ar = Ja+b x?— or x= ( j (a+bf \ («+6)V (8.) Resolved in (Art. 90,) of the Key. (9.) Square the first equation, and x -fy^ia (1) Difference 2x*y* = 12 (2) Subtract (2) from (1) and x— 2x*y*+y = 1 By evolution x* — y* =±1 But x *+y* = 5 2a? 5 =6 or 4. CHAPTER V. (Art. 93.) QUESTIONS PRODUCING PURE EQUATIONS. (5.) Let x-\-y = the greater number, And ...... x — t/= the less Difference • . . .2^=4 Sum =2x 2a?(4#i/) = 1600 Hence. . . .#=10. (6.) Let x-\-y= the greater number. x — y= the less. 2y : x-—y : : 4 : 3 or x= \y. ( x 2 — y 2 )( x—y)=504 (W) (}y) =504 Hence y=4. 3 34 KEY TO ALGEBRA. (7») Let Sx= the length of the field, and 5x= its breadth 40^ x 2 Then =— = the acres. 160 4 ?# 2 X8;r=the whole cost. 26x— the rods around the field. 13X26#=the whole cost. Hence 2x z =lS-26x or #=13 . . . 8x=104,rfns. (8.) Let 5a?— the length of the stack. 4x= the breadth. 7x Then, — = the height. 2 Ix 5x'4x* — -4x= the cost in cents. 2 Also, 5;r-4av224= the cost in cents. Ix Hence, 5x*4x' — •4#=5a: , 4a: , 224 2 7a?-2ar=224 or x—4. (9.) Put x 2 — 7= the number. Then x-\-Jx J +9^=9 ^2-j_9— 9— x or x —4, (10.) Let x represent A's eggs ; then 100 — x— B's 18 8 = A's price. -=B's price. 100— x 18# 8 1 Hence =-(100— #) 100— x x x 9^ 2 =4(100— xf 3#=2(100— x) a?=40. (11.) Let x-\-y= the greater number, x — y= the less. 2#=6 or x=S QUADRATIC EQUATIONS. 35 (x-\-yY=x 3 +3x z y+ 3xy 2 +y* (x^yf=x a -^3x 2 y-{-3xy 2 —y t 2x 3 +6xy 2 =72 Divide by 2x and a*+3y 2 =12 Hence . . »y=l. (12.) Let x= one number Then a 2 x= the other. b aV=6 2 arr=6 x=-. a (13.) Let a? 3 and y 2 represent the numbers. Then a? 2 +i/ 2 =l00 x -\-y = 14 SECTION IV. QUADRATIC EQUATIONS. None of the examples require the aid of a key until we come to the 12th, Art. 101. (12.) Put x=\u. Then 5x 2 = jW 2 , and the equation be comes \u 2 -\-\u= 273 m 2 +4m=1365 Hence w=35 or — 39 which gives . . . x=7 or — 3 j°. (13.) Put x=\u. Then u 2 — 20w =224 W 2__20u-f-100=324 w _10=±18 «=28 or — 8 and • • x=7 or — } (14.) 25# 2 — 20#=— 3. Complete the square by (Art. 99.) 25a; 2 — 20a?+P=/ 2 — 3 5atf=— 10a? t=—2 t 2 =4 5x—2-=^zl x=i or \. 36 KEY TO ALGEBRA. (15.) Put x=^u. Then « 2 — 292u= — 500-21=— 10500 w 2 — 292n4-(146) 2 =108L6 w— -146=±104 w=250 or 42 Hence »=Vt° or 2. (Art. 103.) EXAMPLES. (1.) Put(a?+12)*=y. Then 2/ 2 +2/=6 2/=2 or —3. (a?+12)*=2or — 3 a:-|-12 = 16or81 a?=4 or 69. (2.) Add b 2 to both members and extract square root re then have (rc-f-«)*+&=rt2& The rest of the operation is obvious. (3.) Put y=(9x+4f Then tf+2y=\b y=3 or — 5 Hence 9aH-4=9 or 25. (4.) Put y=(l0-\-xf. Then y 2 — y=2 y=2 or — 1 (Art. 104.) 3 (5.) Put. . . .(x — 5) 2 =?/; then y* — 3^—40 y=S or — 5. (6.) Put (1+37— x*)*=y; then .2 ,,— : 1 2tf From which we obtain y equal 5 or \. Hence . . l-\-x — afej- or ^. From which we obtain x=4t±$jU or izt&.fm. (7.) Put. . . . (a?+ 16)^=1/ ^2 — 3y=io Hence y=5 or — 2. QUADRATIC EQUATIONS. 37 (§.) Puty=a? n ; then d*f—2y=B Put y=\u w 2 — 2w=24 w=6 or — 4 y=Z x= n J2. (9.) Multiply by 4, &c. Rule 2. 4a?*+4a?M- 1=3025 2a?*+l=±55 , a?*=27 or —28 a? s =3 or a?=243. (10.) Put (2a?— 4) 2 =y. 8 16 Then -=H 3/ y 2 8y=^»+16 ^— .8i/+16=0 or y— 4=0. (11.) Multiply by 16. Rule 2. Then . . . 64a?^+16a?*+ 1=39- 16+ 1=625 8a?i+l=25 a?*=3 ;r=729. (12.) Add 5 to both members. Then {x 2 — 2H-5)+6(ar 2 — 2a?+5)*=16 By subtraction, 2/ 2 +6y+9=25 y=2 or — 8. Hence ar 2 — 2a?+5=4 a?=l. (13.) By (Art. 99) we have x* 12a? 1_, 2=/2 _32 361 19 a? 6a? — 1= *=— 6 * 2 =36. 19 19 x 2 12a? 1-36=4 361 19 a? 19 -6=±2 a?=152 or 76. 38 KEY TO ALGEBRA. (14.) Observe that 81 2* and — are both squares, and ar if these are taken for the first and last terms of a binomial 1 square, the middle term must be 9a?*-'2^18. x This indicates to add 1* to both members. Then extract 1 square root 9x-\ — =±10 Hence. • • • x=l or — 1. x (15.) The first member of (15) is the same as (14.) Hence, add unity to both members and extract square 1 29 root ; we then have 0x-\ — = j-4 x x u 93 2 — 43=28 Put 3=- 9 w 2 — 4w=28-9=252 u — 2=±16 3=2. (Art. 105.) (4.) Multiply every term by 3, and ^_83 3 +19a: 2 — 123=0 Operate for square root thus a? 4 — 82?+ 19a*— 12a; (a* — ix 2x*— 4x)— 82?+ 19a? 2 — 83 3 +163 2 33 2 — 123 3(a*— 4x) (a?— 43) 2 +3(3 2 — 43)=0 Divide by (2* — 4a:) and 2*— 43+3=0 3=1 or 3 But the factor 2* — 43 gives 3=0 or 4. QUADRATIC EQUATIONS. 39 (5.) # 4 — 10x 3 +35a^--50^4-24=0 ( x 2 — 5x 2x?— 5x)— 10x 3 +35x 2 jr& OY THE UNlVEESITY 10x 2 — 50*+24 v /y (a^— 5ar) 8 +10(a?»— 5a?)+24^^XrIpQB, , »VJ If we add unity to both members, we shall plete squares. Extract square root, and x*—5x= — 4 or —6 From these two equations we find #=1, 2, 3, or 4. (6.) By mere inspection we perceive that this equation can take the form (a: 2 —- a?) 2 — (a? 2 — #)=132. y2—y=l32 #=12 or — 11. x 2 — x—Vl or — 11 If we take — 11, the value of x will become imaginary, 12 gives #=4 or — 3. ( U. 169. ) (7.) This equation may be put into this form : (y»_ cy)-— 2 (y 2 —cy)=c 2 from which the reduction is easy. (Art 107.) (3.) Taken from the work we have ( a + l)x 2 —a 2 x=a* or (a-\-l)x 2 =(x+l)a 2 Both members are of exactly the same form, and of course the equation could not be verified unless • . a?=«. EXAMPLES. (1.) # 2 + llx=80. Multiply by 4, &c. 4.z 2 -f-./H-ll 2 =320+l21=441 2#-f-ll=±21 x =5 or —16. V\ KEY TO ALGEBRA. (2.) Drop 2x from both members, and divide by 3 ; then x — 1 x — 2 x = #— 3 2 Clearing of fractions and 2X 2 — 6;r— 2x-\-2=x*— 3x—2x-\-6 x 2 — 3*=4 Put 2a=3 Hence, (Art 106) x=4 or I (3.) Multiply the equation by 6x ; then 6x 2 — ■ |-6tf-j-6=13a? x-\- 1 Gx 2 _4-6=7* a:+l 6x 2 +6aH-6=7;r 2 -r-7a: Hence x 2 -^-x—6 x—2 or — 3. (4.) Clearing of fractions 70#— 2 \x 2 +72^=500— 1 50a? 21# 2 — 292*=— 500 (5.) Put f-+y\=x. Then a? 3 -f-a?=30 or, x=5 or — 6. 6 Now, — H/=5 or — 6 y y 2 — 5y=— 6 or t/2+6?/=— 6 Ay 2 —- .#+25=25— 24 2y— 5==hl y=3 or 2. (6.) Put x%=y; then i/ 2 +7#=44 Ay 2 +.#+49=225 2y+7=±15 2/=4or— 11. a?=(4)f or (— 11)*. QUADRATIC EQUATIONS. 41 (7.) x 2 -{-x=42. Hence x=6 or — 7. That is yH-1 1=36 49 or y=5 or «/38. (8.) ar+7 a: x—1 3 33a? — 23 1— 3a?— 2 1 =x*—-lx x 2 — 37a?=— 252 4a? 2 — .tf-I-37 2 =1369— 1008=361 2a;— 37==t=19 *=28 or 9. (9.) 3a? 2 — 9a?=84 12 362 2 — ^+81 = 12-84-1-81 = 1089 Gx— 9=±33. (10.) Clearing of fractions we have 2x+2 t Jx=\6—x ^x-\-2 s /x=\G Multiply by 12, &c. 6^/a?-{-2=±14 a?=2or7j. (II.) 6(2a:— li; a? — 3 3(2a?— 11) •4a?=26 -2a?=13 a:— 3 6a?— 33-f-2a? 2 — 6a?=13a?— 39 2a? 2 — 13a?=— 6 16a? 2 — ^-1-13 2 =169 — 48=121 4x— 13=±11. (12.) Multiply by x* and we have 22a: 2 10a: — \4-\-2x— 9 11a: 2 6a:— 7= 9 11a? 2 — 54a: =—63 42 KEY TO ALGEBRA. u Put x=— ; then u 2 — 54m =—693 11 u 2 —542i-\-21 2 =3Q s u— 27=±6 m=33 or 21. (13.) Clearing of fractions we have £*_! o# 2 -f- 1=* 3 — 6^ 2 ~|-9^— 3x 2 + 1 8z— 27 — # 2 =27.r— 28 tf 2 -f-27x=28 Put 2a=27 a' 2 -f2a^=2«+l x 2 +2ax-{-a 2 =a 2 -{-2a+l a?-fa=±(«+l) • .a?=lor— 28 (14.) Given mx 2 — 2mxjn=nx 2 — rnn to find x By transposition mx 2 — 2mxJn-\-mn=nx 2 Square root . . Jmx — Jmn—dcjnx By transposition {Jm±Jn)x=,Jmn Jmn CHAPTER II. QUADRATIC EQUATIONS CONTAINING TWO OR MORI UNKNOWN QUANTITIES. Problems 1, 2, and 3, require no aid from a key (4.) Put x=vy; then the equations become v*y*+y* = 18vy* (1) vy+y=\2 ' • . . . (2) QUADRATIC EQUATIONS. 43 From (1) we have y— * «M-i 12 From (2) we have y= v-f-1 Hence = v-f-1 v 3 -f-l 3v Divide the denominators by (v-f-1), and 2=—- • Or, 2v 2 — 5v= — 2 v =2 or |. •Another Solution. Cube the 2d equation and we have x 3 -\-ij 3 -\-3xy(x+y) = l2 s That is. . . . 18x'i/H-3xi/(12) = 12-12-12 Divide by 6, and we have 3xy-\-6xy=2'\2-l2 Or 9xy=2'3'4-34 Or ^=2-4-4=32 Now, having (x-\-y) and (xy), the rest is obvious (5.) The first equation can be put in this form (x+y) 2 +2(x-{-y) = l20 Or s 2 -f2s-f-l = 121 *+l=dbll x-\-y=\0 or — 12 Or a?=10 — y or x= — y — 12 From the second equation x= — y 8+y 2 8+v 2 Hence . . _=10— y or, — = — y J 2 y y 8-|-2i/ 2 =:10i/ or, 8+2?/ 2 + 12y=0 y* 5y— 4 ^ 2 -J-6l/= 4 2/ 2 +2«y=2a+l y-fa=±(a+l) 44 KEY TO ALGEBRA. (6.) Put x=vy ; then the equations become vy 2 -\-2y 2 =60 56 y 2 - y 2 =- v 2 -\-v 60 v+2 15 14 Hence = w-f-2 v 2 +v \5v 2 + 15v=Uv-\-28 15v 2 +v=28 4-15 2 t; 2 +^4-l =28-60-)-l = 168l 2-]5v+l = ±41 v=f or— J. (7.) Put x=vy ; then the equations become 6i; 2 i/ 2 +27/ 2 --=5^ 2 4-12 (1) 2ut/ 2 -f3i; 2 ?/ 2 =3i/ 2 — 3 (2) 12 From (1), . . . y 2 = — ■ — —3* From (2), . . . y 2 3^2 4-2^—3 4 1 Hence . . . j =0 6u 2 — 5v+2 3?; 2 +2v— 3 12v 2 +8v— 12+6t; 2 — 5u+2=0 18v 2 +3i>— 10=0 9v 2 -t-|v=5 Complete the square, (Art. 99.) §v 2 -\-\v-\-t 2 =t 2 -\-5 3vt=lv or, t=% * 2 = T V 9v 2 4-Iv+T , a=T , 6+5=fi 3i;-f-*=±} v =f r— f, (§.) Put x=vy ; then the equations become 3v 2 y 2 +vy 2 =68 (1) 4?/ 2 -f-3u2/ 2 = l60 (2) * QUADRATIC EQUATIONS. 45 68 160 3v 2 +v " 4+3i> 40 17 4+3v 'Sv 2 -\-v 120v 2 +40v=51v+68 120v 2 — llu=68 4-(120) 2 ?; 2 — .tf+121=68-480+121=32761 2-120v— ll=rbl81 u=| or— jj. (Art. 111.) (2.) From the second equation we get x*y*=6 or, 2aA/* = 12 This added to the first equation gives Put x*-\-y*=s; then s 2 +2s=35 . . . s=5 or —7. 1 1 Now if we put a? 3 =P, and y* = Q, we shall have P++#=26, and P#=8 2jPQ =16 (P+£) 2 +(P+£)=42 Hence . .P+£=6. 46 KEY TO ALGEBRA. i £C* 33 3 11 (5.) Put — =u ; then u 2 -\-4u— — . . u=- or . yk 4 2 2 The remaining operation is obvious. (6.) Given # 2 — 8# 2 y-64, and y—2x*y*=4, to find a; and y. To both members of the first equation add 16#, and to the second add x, to complete the squares ; then extract square root, and we have y— 4ar*=4(a?+4)* and y*— x* = (x+4)* Four times the last equation subtracted from the prece- i ding gives y — 4i/ 3 =0 or y=16. (7.) Multiply in the first equation as indicated, and sub tract the second equation ; we then have x+y+2x*y*=25 or x*-\-y*=5 But from the second equation we have (•*-+y*)**y* a - 80 Hence #V= 6 - 3 3 1 (8.) Divide the first equation by y 2 , and x 3 =2y*, or y =kx 3 This put in the second equation gives Sxt— £a?l=l4 al—16#*+64=64— 28=36 CHAPTER III. QUESTIONS PRODUCING QUADRATIC EQUATIONS. We pass to the sixth. (6.) Let t= the time (hours) he traveled, and r= his rate per hour; then rt=36 (1) QUADRATIC EQUATIONS. 47 But if r becomes (r-J-1), t must become (t — 3), and then (r+l)(/— 3)=36 . . . . (2) Or rt-\-t— 3r— 3=36 rt =36 *.=3(r4-l) Hence r 2 -f-r=12, and r=3. (7.) Let x= the number of children, And • • • y= the original share of each. Then . . . a?y=46800 (1) (a?— -2)(i/+1950)=46800 (2) a?*/-f 1950a?— 2y— 2- 1950=46800 1950(*— 2)=2i/ Or 975(a?— 2)*=a? f */=46809 By division, 3* — 2a?=48 #=8. (8.) Let a?= the number of pieces. 675 Then = the cost of each piece. x 675 48^. =675 x 48a? 2 — 675a*=675 16a? 2 — 225a?=225 ^9.j Let a?=the purchase money. 104a? 104a? Then = the cost, and 390 = his whole gain. 100 100 6 104a? 104a? a? Then : 390 : : 100 : — 100 100 12 Product of extremes and means 26a? 2 =39000— 104a? 300 2a? 2 =3000—8a? 300 48 KEY TO ALGEBRA. Put a=300 and divide by 2 ; then # 2 — =5a — 4a? a # 2 -J-4a#=5a 2 x 2 +4ax-\-4a 2 =9a* #-J-2a=3« #=0=300. (lO.) Put x-\-y— the greater part, And • • . x — y = the less part. Then 2#=60, #=30, and x 2 — i/ 2 =704. (11.) Let x— the cost ; then 39 — x= the whole gain. x : 39— x : : 100 : x Ans. a?=10. (12.) Let {x — 20)= the persons relieved by B. Then . • . #-f-20 = the persons ....... A. 1200 1200 + 5: ar-f-20 #—20 Divide by 5, and put «=240 ; then a a 1=- #+29 x— 20 o #__20a-f a?— 400 =ax-\-20a # 2 =4O0+4OO=4O(«-|-1O)=4O-25O Or # 2 =400-25 #=20-5=100. Hence 80 is B's number, and 120 A's. (13.) Let #= the price of a dozen sherry, And y= the price of a dozen claret. 7x-\-l2y=50 (1) 10 — = the number of dozen of sherry for ^10 # 6 -=the number of dozen claret for £6, y 10 6 Then . . . — =3-f- * (2) # y Or QUADRATIC EQUATIONS. 4tf 10 10y 70y By substitution, --- {-12y=50 70t/+36?/ 2 +72?/=150?/+50'6 36*/ 2 — 8t/=300 9# 2 — 2y=75 Hence y=3. (14.) Let 19#= the whole journey. Then x= B's days, also his rate per day. Or x 2 = B's distance. Also, 7#-{-32 = A's distance. x 2 -|-7aH-32 = 19# tf2__12#=— 32 Hence #=8 or 4. And 19a?=152 or 76. If we put x for the whole journey, we shall obtain the 13th equation, (Art. 104.) (15.) Let x= the bushels of wheat, And . . . #-f-16= the bushels of barley. 24_ 24 1 x a?+16 4 x 2 + \§x 24*-f 16-24=24aH 4 a? 2 -f-16*=16-96=16-16-6 Put 2a=16. Then 2a-2«-6=24a 2 x*+2ax=24a 2 x+a=±:5a #=4a=32. (16.) A put in 4 horses, and B put in x horses. 18 Then — =the rate per head. x 4 50 KEY TO ALGEBRA. 4-18 J— 18== the price of the pasture. x 4-20 __— }-20= the price of the pasture. x~\~2 4-18 4-20 Hence .... = \-2 x a?-f-2 36 40 — =-3-:+l *=6 x x-\-2 (17.) Let 4x= the price per yard, And . . . 9x= the number of yards. 36a? 2 =324 x =3. (18.) Let \0x-\-y= the number. Then Z£=2 (1) xy And 10a?+y-r-27==10y-r-a? (2) From (1). . . . 10a?=(2a?— \)y From (2) • • • • x-\-3=y ^ ,. . . l0x By division, . . =2x — 1 ir-f-3 1 0a7— 2a? 2 + 6a?— y=v-H=«=J±$«/ — 3 (21.) Let x-\-y= the greater number, And .... x — y= the less. ^^ 7 ^24 (1) 2x+2x 2 +ty 2 =62 (2) Or x+x 2 +y 2 =3l Add (1) x 2 — y 2 =24 2x 2 -\-x =55 #=5, y=l. (22.) Let x-\-y= the greater number, And . - . x — y= the less. **— 2/ 2 +2x=47 (1) 2x 2 +2y 2 — 2#=62 (2) 2x 2 — 2?/ 2 -r-4a?=94 4^-f2x= 156 2x*+ x= 78 x=6, y=l. (23.) Let (a?+y)= one number, And .... (x — y)= the other. 2z=27 (1) x*+3x*y+3xy*+y* X s — 3x*y -f- 3.ri/ 2 — t y 3 2x 3 +6^ 2 =5103 (2) Divide (2) by (1), and we have **-f-3i/ 2 =189 52 KEY TO ALGEBRA. 3 2 -9 2 \-3y 2 =9-2l 3*81 Divide by 3, and f-y=3-21 4 3-81-j-4t/ 2 =3-84 4?/ 2 =3-3 or 2y=3 y==|. *-h/=v+f=i5, &c. (24.) Let #+y= one number, And .... x — y= the less. 2x= 9 x 4 -\- 4x*y-\- 6xy-\-4xf-\-y* x 4 — 4# 3 2/+6#y— 4ay-r-y 4 2a? 4 +12a?y +2^=2417 By resolving this we shall find y=\. (25.) Let #+?/= the greater, and x — y= the less. Then O^ 2 — 3/ 2 ) (2# 2 -f 2*/ 2 )=1248 (1) Or x 4 — -y 4 =624 Also 4a??/=20 (2) Or y=- x x 625 625 a? 4 — =624 «4 X* # 8 — 624a? 4 =625 Put 2a=624 Then x*—2ax 4 -\-a 2 =a 2 -\-2a+l x 4 — «=±(«+l) a? 4 =2«-f-l or — I a?=5 y=i. (26.) Let a?= the days required by one, And .... #+10= the days required by the other. ARITHMETICAL PROGRESSION. 53 Then . • . -= one day's work of the .first. x = one day's work of' the second. #+10 J x #+10 12 SECTION V. CHAPTER I. ARITHMETICAL PROGRESSION. (Art. 116.) (4.) Z=l+3kZ (A) 280=16(24-31)^ (B) (B) reduced gives (/=£ ; then (A) gives Z=16£. (5.) Here a=|, L=h n=5. *=l+4d (A) *=(*+«i (2*) From {A) we have d=£ T 5+2^=5= the first mean, &c. (6.) Here a=9, Z=109, n=ll. J09=9-f-10rf )~',we take formula (3). Then a=a, x= — 6, and m— — 1. Hence, a m +ma m -'b-\-mPj a m ~ 2 b 2 , &,c.=a~ x +a--6+a" 3 6 2 , &c. " 1 b b 2 Or -+-7+-T' &c - a a z cr b 2 (4.) Take formula (1), and put x= — . Then a 2 K J \ 2 2 22 3 / / b 2 b* 36 8 \ a ( H H &c. ) \ 2a 2 2-4a 4 2-46a 6 / (5.) Expand = -=-( H ) («•+*•)* c(n-gy A c2) The numerical coefficients of this series must be the same as the last, because m is numerically the same ; the sign of the second and every alternate term will be minus. d/ x 2 3x 4 3-5x 6 3-5-7.T 8 \ -( 1 H -f- Ac. ) c\ 2c 2 2-4c 4 2-4-6c 6 2-4-6'8c 8 / , X3 / « 2 \? |/ # 2 \f a 2 / a*\f («•) (*)'0— ) =<<'—) =7 ( 1 ^) 3 a; x 2 Here m= 4 a a 2 x m — 1 x 2 Formula (2) 1+m — \- m- &c. = w a % a 2 / 3^ 3 a* 4 3 5* 6 \ ( 1 926179 250 2324837 1852358 « 926429 51 4724793 4632145 #=463251. 926480 926480 926480 As the algebraic sum of the two roots must make 21, (Art. 156 in the work,) therefore the other root must be —463230. (9.) Given 7x 2 — 3#— 375, to find one value of x. Or # 2 — ?#= 3 I 5 . Putx=\y. (Art. 166.) Then y 2 Sy 375 Or I/ 2 — 3y=375X 7=2625. 49 49 7 In this equation we perceive that y must be more than the square root of 2625, that is, more than 50. Hence put r=50. rs t — a-\- r . . . . .47 2625 ( 52 r-f-s 52 235 — a+2r-fs. . . 99 275 s+t 2.7 198 1017 7700 u 75 7119 H 10245 58100 51225 6875 52.756-f- „ , Hence x= =7.+ EQUATIONS. 69 (10.) Given 2x* — lla?= — 7|, to find one value of x. Orx 2 - 'Jar==— y Put a?=£y. Then . . . ly 2 — y y=— y or. . .# 2 — lli/= — 15. In this equation one value of y is between 9 and 10 ; therefore put r—9. — a+ r —2 —15 ( 9.405124838+ r-\-s 9.4 —18 — a-f-2r-f-s 7.4 300 *+* 4 296 7.805 40000 51 39025 78101 97500 12 78101 781022 1939900 24 1562044 7810244 377856 The division is not carried out for the last three figures. Hence one value of y is 9.405124838-J-, and as the two values must make 11, (Art. 166), the other value must be 1.594875161. But x=iy, therefore #=.797437580+ or, #=4.7025624194- Am. (11.) Given |# 2 +|a?= T 7 T , to find one value of x. Or # 2 -f-ia:=!f or, ar 2 +.8#=0.848484848+ Here it is obvious that x cannot be 1 ; by trial we find it must be near .6 ; therefore r=6. a+r 1.4 0.8484848484 &c. ( 0.604233+ r+s 60 84 2004 8484 42 8016 20082 46884 23 40164 200843 672084 70 KEY TO ALGEBRA. We omit several examples as they present no difficulty. CUBIC EQUATIONS. (3.) Given x*-{-2x a — 23#=70, to find one value of x. By trial we find x must be a little over 5; therefore r=5, .#=2, ,#=—23, N=70. B —23 r(r+A) 35") 1st Divisor 12 > 70 ( 5.134 r 2 . . 25J 60 B' 72 10000 *(a+3r+^) 17n 7371 2d Divisor 7371 ^ 2629000 s 2 1J 2276697 B" 7543 352303000 *(3Q+t)t 4599 305649104 3d Divisor 758899 46653896 f 9 B" 763507 61576 4th Divisor 76412276 16 Common division will give three or four more figures to perfect accuracy. (4.) Given x s — 17^+42^=185, to find one value of x. Here .#=—17, 5=42, JV=185, and we find by trial that x must be between 15 and 16 ; therefore r=15. * Q represents the root as far as previously determined. CUBIC EQUATIONS. 71 B 42 r(r+Ji) —30^ I r st 1st Divisor ... 12 > 185 ( 15.02 r 2 . . . 225J 180 B' 207 5000000 *(s-f3r-M) .... 0") 4154008 2d Divisor 207 f 2077 ) 845992 ( 407 s 2 OJ 8298 B" 207 16192 t(3Q-{-t) 7004 14539 3d Divisor 2077004 1653.0 Hence x=15.02407-|- (5.) Given x 3 -{-x i -^500 i to find one value of x. Here Aml % B=0, r=7. B r(r-\-Jl) 56 1st Divisor .... 56 500 ( 7.61 r 2 49 392 B' 161 108 (3r+*-M)s . ■ • • 1356 104736 2d Divisor .... 17456 ■ 3264 * a 36 1887181 18848 )1376819 2381 Continue by common division. 3d Divisor .... 1887181 1 1889563 (6.) Given # 3 +10x 2 -j-5tf=2600, to find one value of x. 72 KEY TO ALGEBRA. Here ^=10, B=5, r=ll. B 5 2600 ( 11.006 r(r+^). ... 23 n 2596 1st Divisor . . ■ 236 V 4 r 2 121J 3529188216 B' 588 470811784 (3/?+tt)w. • • • 198036 . , t. . . ~ nntnnn ^ n Continue by common division. 4th Divisor . . . 588198036 J The four miscellaneous examples on page 262, the last page of the school edition. (1.) Let x-\-y represent the greatest extreme, and x — y x 2 — y 2 the least extreme ; then — — = the harmonical mean. x By the conditions of the problem we have , x 2 — v 2 (x 2 — V 2 ) 2x+ -=26. • . .(1) and * —=576 . . . (2) x x By reducing (1), and taking the square root of equation (2), we have 2x 2 +x 2 — y 2 =2Qx (3) x 2 —y 2 =24 ) Jx~ (4) By subtraction . . . 2x 2 = 2Qx — 24 Jx If we put jjx=v, and reduce the resulting equation, we shall have »*— 13t>=— 12 By (Art. 163) we find v=3. -Hence a?=9, &c. (2.) Let x-\-y= the greater number, and x — y= the less. Then by the problem we have x=5 And 8^1/4- 8.n/ 3 =1040 Or y*+25y=26 Hence y=l (3.) Let x, y, and z represent the numbers ; then by the problem x-\-y-\-z=2S (1) x+yz=5l (2) y+xz=S7 (3) EQUATIONS. 73 By adding (2) and (3) we have (x+y)-\-(x+y)z=l38 But by equation (1), x-\-y =28 — z ; therefore 28— s+28z— z 2 =138 or z=5 (4.) Let x, y, and z represent the numbers ; then ^+2/ 2 +z s = 195 (1) x 9 +y 9 +z 3 =l799 (2) xyz= 385 (3) The form of these equations will not be changed by ta- king x for z or x for y. A full and formal solution of these equations would become tedious, and to avoid this, we may venture a solution by inspection. As 195 and 385 both have 5 in the unit's place, it is more than probable that the value of one of the symbols is 5. Therefore assume z=5. This gives ic 2 -f-i/ 2 =170, and #*/=77, from which we find a?=7 or 11, and y=ll or 7, and as 5, 7, and 11 will verify equation (2), this is a true solution. The following examples are in the University Edition only. (6.) Find one value of x from bx*— 6x 2 -\-3x=— 85. As the result is negative, we will change the second and every alternate sign of the equation, (Art. 178), and find a value of x from the equation 5r J -f-6x 2 -r-3a;=85 Use the formula of (Art. 194.) c=5, J3=6 y J5=3, and by trial we find r=2. r s B 3 85 (2.1 (cr-f-^)r .... 32V 70 1st divisor . . 35 f 15 cr* 20J 9.065 87 5 935 * ' * * Continuing this we shall find the 2d Divisor . • • 90.65 value of x to be 2.16399-f , and its cs _^ sign changed will be the value of x 94.35 in the original equation. 74 KEY TO ALGEBRA. (7.) Find x from the equation \2x*-{-x 2 — 5#=330 Here c=12, J2=l, J5=— 5. r=3. £ —5 (cr+A)r lin I r »< 1st Divisor .... 106 ^ 330 ( 3.036 cr* 108J 318 B' 325 "~12 {3cE-\-ct)t. . . • 11208 9783624 3d Divisor. . . . 3261208 2216376 ct 2 108 Continue thus. 3272524 In the same manner perform (8) and (9.) ( Art. 195. ) Page 323. (3.) Extract the cube root of 1-352-605-460-594-688. For the sake of brevity, take r=ll, in place of 1. 1st Divisor. • -121 r st JS'^r 2 . • 363 1-352-605-460-594-688 (110592 (3B-\-t)t. • 16525 1 331 2d Divisor 3646525 21 605 460 25 18 232 625 3663075 3 372 835 594 (3R+u)u • • 298431 3 299 453 379 3d Divisor 366605931 73 382 215 688 81 73 382 215 688 366904443 (3i?-f-v)u • ■ 663544 36691108844 (4.) By a table of cubes which run to 8000, we perceive at once that r in this example is 17. EQUATIONS 1st Divisor 289 Sr 2 =B' 867 (3#-fs)s 2575 2d Divisor 89275 25 (3B+t)t 9J875 10504 rs t 5382674 ( 175.2 4913 469674 446375 23299000 18396008 75 3d Divisor 9198004 4902992 4 Complete another divisor, 9208512 t * ien contmue as m simple di- It is not important to show a solution to the remaining examples under this article. Page 324. (»•) Change all the signs, then a*+a*+3?— £=500 r=4 r\ i 1 —1 500 (4 4 20 84 332 5 2l 83 1~68 c 4 36 228 9 57 311 go 4 52 b Ti To9 • 4 _ 8 1 17 109 311 168 (.4 =l± > /5 (7) Again: Add 2 to each member of (5), and we have v-\-2=v 2 -\-l. Also, multiply (5) by v, and v 2 =v 3 — v y or # 2 ~j-z;=fl 3 . But v 2 =v-\-\\ Whence, . 2v-\-\—v 3 , and 2v=v 3 — 1. Now, Equation (6) becomes 2vy=v-\-2 y or 4vy=2v-^-4. Substituting the value of 4v and 2v, as found in (7), we have 2(\±j5)y=5±j5 2y= 5/y±T =±V5 ******* Ans ' Lastly, x^vy=^ (\d-j5)^ s /5=\ (J5±5) Arts. EQUATIONS. 79 Another solution may be found in our Mathematical Operations, pages 115 and 116. (Art. 204.) Example 1. x*—5x 3 +5x 2 —l=0. Here the sum of the coefficients is zero, therefore one root is -f-l> and the equation is divisible by x — 1. Dividing by (x — 1), the quotient is x *-\-x 3 —4z 2 -\-x-{-l=:0. Here, again, the sum of the coefficients is zero, show- ing that another root is +1; and this, again, is divisible by (x — 1). The next quotient is x 3+2x 2 — 2x— 1=0. Here, again, the sum of the coefficients is zero, show- ing a third root equal to -(-1; and hence, we divide again bj x — 1. The quotient is s*_|-3;z+l=0. Whence, x= — \ (3±J5). N". B. — We might have taken x*-\-x 3 — 4# 2 -|-£-|-l=0, and solved it by the rule (under Art. 203). 2. x*+5x 3 ~\-2x 2 ±5x-{-l=0. By the rule (under Art. 203) we have x< -j_5z 3 -f( 2+V )x a -j-5^-1-1 = yz 2 ; Extracting square root, and x 2 =— 1, or x=±J—l; Or, x 2 +5x=— 1 *=H— 5±J2l~ 3. x*— 4x 3 -\-4x— 1=0. (a.*— 1)— 4.r(z 2 — 1)=0. Dividing by (x 2 — 1), and x 2 -\-\ — 4^=0, or x 2 — 1=0, Or . . #=d=l; or x 2 — 4.r-)-4=3. #=2± A /3. 4. x*—^x 3 -{-2x 2 — \x -(-1=0. By the rule (Art. 203), add \\x 2 to each member— then *«— lx 3 +\\x 2 — fz-f^ffz 2 Square root, > x 2 — |-ar+l ==fc|ar, 80 KEY TO ALGEBRA. Whence, . . # 2 -|-l=0, or x 2 — fa--|-l=0 x=±z,J — I, or x=2 or £. 5. 5x*-\-Zx 3 +9x 2 +8a--f-5=0. Add .... 2a-- 2 =-|-U>a; 2 and # 4 +fa: 3 +2a: 2 +fav}-l = ?L Add ISfl =1^1 25 25_ Square root, x 2 -\-$x-\-l=±xj2\ a quadratic. Place .... 4 ~-^ 21 =2fl 4— 721=10a, 5 Then .... s 2 +2az+a 2 =a 2 — 1 a?= — a-±ija 2 — 1 Because a 2 = — _5l — / a 2 j is imaginary, and, therefore, the values of x are imaginary. 6. 4a* 6 — 24a? 5 +67a: 4 — 73a- 3 +57a- 2 — 24a-+4=0. Divide by x 3 ; then 4a: 3 — 24ar 2 +57a;— 73+^— ^4--i-==0 (1) XXX 1 ^7 Assume a--|--=y, Then 57a;+ — =57y, a; a; a: a +2+-L=y 2 — 24z 2 — — =48— 24y s a; 2 a; 2 Also, a- 3 +-U-3 (x-\S\^y\ or a-^*^ 3 — 3y, # 3 \ a?/ * 4 and 4# 3 4- — =4y 3 — 12y. x 3 These values placed in ( 1 ) And 4y 3 — 12y+48— 24y 2 +57y— 73=0 EQUATIONS. 81 Or 4y 3 — 24y 2 -f-45y— 25=0, (2) Here the sum of the coefficients is 'zero. Therefore, one value of y is 1. Then &4-!=l. Whence x= l±z ^~^- 1 x 2 Dividing (2) by y — 1, and we obtain 4y 2 — 20y+25=0 Square root 2y — 5=0, or y=f. Whence #-!-_=§, and #=2 or £. x 7. 4j? 4 +3« 3 — 8a; 2 — 3ar+4=0. (1) Here the sum of the coefficients is zero. Therefore, #=1 for one root. Again, if we change the second and every alternate sign, we shall have and the sum of the coefficients is still zero; therefore, x=l in this equation, which corresponds to — 1 in the given equation. Therefore, (1) has two roots, x=\ and x= — 1: hence, that equation is divisible x 2 — 1. The quotient is, 4# 2 -|-3a; — 4=0, Whence x=~ 3±z 'J 13 8 8. x*+24x 3 — 2z 2 — 24z-fl=0. This, like the preceding, and for the same reason, is divisible by x 2 — 1. The quotient is, # 2 -f24.r — 1=0, Whence *==■— IZdzjUB. 9. x *—2x*—7x 2 —Sx+\6=0. (1) Here the sum of the coefficients is zero. Hence, a?=l for one root, and the equation can be depressed to x 3 — x 2 — 8z— 16=0. (2) Assume x=nP. Then n s p*—ri> p3_ 8w p_! 6==0 Letrc=4; then 64P 3 — 16P 2 — 32P— 16=0 Divide by 16, 4P 3 — P 2 — 2P— 1=0. 82 KEY TO ALGEBRA. Here the sum of the coefficients is zero. Therefore, P=l. But x—?iP, and n=4, P=\. Whence, rr=4, for another root of the equation. Now (2), divided by x — 4, produces ar 2 -j-3:r-J-4=:0, "Whence x=— — -V Imaginary. Another Solution: The attempt to extract square root results as follows: #4_2z 3 — lx 2 — 8z-j-16 (x 2 — x x* ^2x^~—7x 2 2x 2 —x — 2x 3 -\- x 2 — 8x 2 — Sx If the remainder were -\-8x 2 in place of — 8# a , this ex- pression would be a square. It will be 8x 2 , if we add -{-\6x 2 to each member. Then we shall have x x — 2x 3 -\-x 2 -\-8x 2 — 8x+16=\6x 2 , Or . . . (x 2 — x) 2 +8(x 2 — x)+\6 = 16x 2 , Square root x 2 — x-\-4=zh4x. 10. x*+2x 3 —3x 2 —4z+4--=0. (1) Here one value of # is 1. Dividing by x — 1, we obtain x 3 +3x 2 — 4=0. Here, again, x=l; and another division produces Square root . . (#+2) (z+2)=0, x=— 2 z=— 2. 11. x*— 2x 3 — 25.c 2 +26.r-f-120=0. (1) Assume x=nP. Then n i P<—2n 3 P 3 — 25w 2 P 2 -f2GttP+ 120=0 p4 _2P3_25 yj2+ 26 p+ i20 ===()! n n 2 n 3 n l Letw=3. Then P^—fpz — ^pz+^P+^^O, Or 27P 4 — 18/ J3 — 75P 2 +26P+40=0. Here the sum of the coefficients is zero; therefore, P=\. But n=o, whence x=3. EQUATIONS. 83 If in (3) we assume n—5, the coefficients will again be zero, showing that another root is 5. Now, equation (1) divided by x — 3 or x — 5, or thei r product, will produce a quadratic whicb^fl^gi\]e r towp\ other roots, #= — 2 and x= — 4. Jy^^ of the ^ 12. r 4 — 2x 3 +2x 2 — x^ofm N I V E EjCSDX T Y (x 3 — \)x=2x 2 (x — 1V^ - ^ As x is a common factor, x=Q; an 3^ft^Jl 7^(^60^ ■»**• common factor, therefore' x=\. Dividing tors, we obtain x 2 -]~x-\-l = 2x, Or x *—x=—\. Whence x^^^" 3 2 1 3. x 4 —4x 3 -\-8x 2 — 32a-=0. ( 1 ) Or (x— 4)x 3 +{x— 4)8#=0, Whence z=0, Or . . x=4, or # 2 -|-#=0, or x=dtzj — 8. 14. x 3 +5x 2 +3a?— 9=0. (1) Here the sum of the coefficients is zero; therefore, x=l. And by division we obtain £2_|_6:r+l=0, Whence #=—3^78. 15. o;3_|_ 6 , r 3_ 7x _ 60=0. (1) Assume x=nP. Then n 3 P 3 +6n a P 2 —7nP—60=0. Suppose n=S. Then 3)27P 3 +54P 2 — 21P— 60=0 9P 3 -{-\QP 2 — IP— 20=0. Here the sum of the coefficients is zero; therefore, x=S, for one of the roots o^pthe equation. The other roots are — 4 and — 5. 16. ^ 3 -[-8^ 2 + 17^4-10=0. (1) Change the 2d and each alternate sign, then x 3—8x 2 _f_17#— 10=0. Here the sum of the coefficients is zero. Hence, x=\ for the last equation; or, x= — 1 for the given equation. Now divide the given equation by *-|-l, and the quotient will be x 2 4-7x-\-\0=0, x=—2 or —5. 84 KEY TO ALGEBRA. 17. x 9 —- 29x 2 -f 198z— 360=0. Place x=?iP. Then p 3 4,V [ . 198 P 360_ Q JT — | n n 2 n* Assume w=3. Then (') p* _?£p 2 J_22P —12=0 3 3 3/?3_29 J p2^_ 66 p_ 40=0 Here the sum of the coefficients is 0. Therefore, x=3; and, dividing the equation by (x — 3), we obtain x 2 — 26*+ 120=0, x=6 or 20. 18. 4x 3 — 112z 2 + 109ar— 27=0. (1) Here the absolute term — 27 is not numerically large enough to make the sum of the coefficients zero; there- fore, we take an operation which will increase it: p Place x= — Then 2 4P 3 112P 2 , 109P •27=0 8 4 ' 2 Multiply by 2, and P 3 — 56P 2 +109P— 54=0. (2) Here the sum of the coefficients is zero. Therefore, P=\. Whence x=\. Dividing (2) by P — 1, we obtain P 2 —55P4-54==0. Here, again, P—\; and, of course, another root of the original equation is \, and it has two equal roots. Dividing (1) by x — \, a#d we obtain 4x 2 — 110^+54=0, Whence .... #=27 ON INDETERMINATE EQUATIONS. For the complete solution of a problem, we must have as many independent equations as unknown quantities to be determined. When this is not the case, the problem is indeterminate. For example, x-\-y—20. x may be one or two or three, or 5, or any other number, whole or fractional, under 20, and y will take the remaining part of 20, and the equation is indeterminate in the strictest sense of the term. If, however, we restrict the values of x and y, to whole or integer numbers in the equation rc-f-?/=20, x cannot have more than 20 different values, when, without this restriction x might take an infinite number of values, and still preserve the equation x-\-i/=20. In some cases, the number of answers to an equation may be infinite, and the particular values restricted to inte- gers. The following is a general case of the kind ax — by—c. This givi s x= —£— : , where y may be any a value whatever, that will give ' , a whole number, but Cb numberless such values of y can be found, as c-\-by is a magnitude that can rise higher and higher, without limit, according to the assumed value of y. 86 KEY TO ALGEBRA. But take y what we will, and the equation still exists, and therefore, the member of ansivers for x is unlimited or infinite. In such equations, if the least values of x and y are required, we have definite problems. In equations like the following, ax-\-by—c, the number of answers in integer numbers, may be very limited, may be only one, or may be impossible. The equation gives a Now, if c is very large, and b and a small, y may take many different values before c — by is so small that we ean- not divide it by a, and obtain an integer quotient. When c is not large in reference to b and &, we may ob- tain only one value of y and #, and if by making y=l, we c find — -, a proper fraction, the problem is impossible. a 3x-\-5y=\3. a ?= ' Q ° y . If we take y—l. x=-^, not an integer, therefore, y must not be taken equal to one. Take y=2, then a?=l, both integer values, and the only integer values that will answer the conditions of the equa- tion. 3#-)-5^=6, is an equation in which it is impossible to give integer values to both x and y, because 3— 1-5 is more c—b . , . than 6, or a-\-b greater than c or is a proper fraction. The equation az-\-b?/=c. is always possible in integers, when c is greater than (ab — a — b). and a and b prime to each other. The equation is sometimes possible, when c is not greater than {ab — a — h.) The equation lx-\-l3i/==7\, is impossible in integers, for both x and y. But the equation lx-\-\3y=27. is possible, a? =2 and y=l. Here a and b are the same in both equations. , In the first equation, e=71 ; c is not large enough to make the equation possible, for c, 71, ia y INDETERMINATE' EQUATIONS. 87 not greater than (7*13 — 20), but if c was any number greater than 71, the equation would be possible -in integers, and it is possible, with some numbers less than 71. If a and b are not prime to each other in the equation ax-^-by=c, c must be divisible by the same number that di- vides a and b, or the equation is impossible in integers for x and y. Examples. 1. Giv en 6a-f9y=32. O r, 2a?-f3//=y. But if x is a whole number, 2x will be a whole number, and by the same considerations, 3y must be a whole number, and two or more whole numbers added together can never make a fraction, therefore the equation 2x-^-3y= 3 f , or 6r-|~9?/=32, is impossible in integers. In cases where solutions are possible, our rules of opera- tion rest entirely on these considerations: 1st. A whole number added to a whole number, the sum is a whole number. 2d. A whole number taken from a whole number, the remainder is a whole, number. 1 3d. Multiply a whole number by a whole number, and the ■ product is a whole number. For instance, if x is a whole number, 2.r, 3a?, 4x, or any integral number of x is a whole number. 2. Given 3 a?-f»5y=35, to find x and y in whole num- **»■ i — | I W r * "" bers. a?= Q But as x is a whole number, its equal, o 35— 5» or — q~^i must equal some whole number. Now 11 — y being a whole number take it away, and * the remainder — -— , must also be a whole number. o 88 KEY TO ALGEBRA. Sv But y being a whole number, -- is a whole number, 3 ml 3y , 2—2;/ ?/4-2 . . Therefore, - -j — -= — =— ^— is a whole number, which number call ;?. And - '- ==p. Or, y=3p — 2. For the least value of y make p=l, and ?/ will equal 1, 35 5„ anda?= — = — =10. Make p=2, then ?/=4, and #=5. 3 Make ^=3, then ?/=7, and a?=0. Hence, ?/=l or 4, and z=10 or 5, are the only results this equation admits of In operating on the fractional expression — - — *, it was 3 our object to work down the coefficient of y to 1 . To accom- plish ti>is object, we cast out whole numbers, add and sub- tract whole numbers in the shape of fractions, &c, only- taking care to keep expressions that are equal to integers, until the coefficient of y becomes one. 3. Given 35a? — 24?/= 68, to find the least values of # and y m integers. The number of values in this case is unlimited. _68+24?/_ 33+24?/ *- 35 - 1+ ~35" Hence, — ~J~- — - = some whole number ; but -~ is also 3d 3d Soy 33-4-24// Ut/ — 33 a whole number, -p- ' - • = -^ — =an integer. 3d 3d 3d ° 33 j /-99_33//-29 Ur ' ""35 ~ 35~ 35?/ 33?/— 29 27/-J-29 _ .■ . -h^ qcl~ = qc — a wnoIe number, multiply by 18, 3d 3d 3d . 36?/+18-29 , % . , y+32 . . and — ' ■ — =3/-f-14-f-- - ' - = a whole number. 3d 3d Therefore V —^-=p. Or, y=Sbp— 32. INDETERMINATE EQUATIONS gg Take p=l for the least value of y. and y—3. Therefore *=4. 4. A man wishes to lay out $500 for cows and sheep: the cows at 17 dollars per head, and the sheep at 2 dollars. How many of each did he purchase 1 Let z== the number of cows, and y the number of sheep. Then 17#-|-2?/=500. We know this equation is restricted to whole numbers ; because the man could not have part of a cow, or part of a sheep. To find the least number of cows, transpose 17a?. Then y = 250— 8*— ^. * 2 Now as y must be a whole number, and 250 — Sx must x also equal some whole number, must be a whole number. That is, the number of cows must be an even number ; be- cause the number must be divisible by 2. x Hence, -^=P- Or, x=2p. Makep=l, and a?=2, the least number of cows. Then #=233, the corresponding number of sheep. Now if the man wished to purchase as few sheep, and as many cows as possible, we should transpose the other term, 500— 2y on , 7—2?/ thus: * = ~ 17 = + ~T7 ' 7 2?/ Therefore, — =— -- = a whole number. Multiply by 8, and — — — - = a whole number, to which add — -±- and we 17 17 have — ~^=3-| — ^. Drop off the whole number 3, then 5 I y -J^L=zp. Or, y=l7p — 5. Making p=l, gives #=12, the smallest number of sheep. This gives #=28, the cor- responding number of cows. The number of cows or a?, may be any one of the even numbers from 2 to 28. a 90 KEY TO ALGEBRA. 5. A man wished to spend 100 dollars in cows, sheep, and geese ; cows at 10 dollars a piece, sheep at 2 dollars, and geese at 25cts., and the aggregate number of animals to be 100. How many must he purchase of each ? Let x=. the number of cows, y the sheep, and z the geese. Then - - - 1 Qx+2y+ | = 100. (1) And - - - - x-\- ?/-|-* =100. (2) Clear equation (1) of fractions, and 40*+8#- r -;?=40G. , T _j_ y _j_ 2 r == l00. 39x+7y =300. 300—7?/ „ , 27—7?/ , , . xz= — ' '— H ^- 1 =a whole number. Or, 5( S9 j^= — 3 ^— - = a whole number, add -^ , 4y+135 40* -f- 1350 . ?/+24 , . and Kg- , or ^^— = ?/+34+ tjjj- = a whole number. Therefore, 8tt-~ ^ Or, ?/=39^— 24=15. This value of y, gives a?=5. Hence, z=80. If we take p=2, we shall have y=54; then a? will come a minus quantity an inadmissible circumstance in any prob- lem like this. Therefore, 5 cows, 15 sheep, and 80 geese, is the only solution. 6 A person spent 28 shillings in clucks and geese; for the geese he paid 4s. 4d. a piece, and for the ducks, 2s. 6d. a piece. What number had he of each? Let #=the number of geese, and ?/ the number of ducks. Then. 52.r-f30.?/=28-i 2. Or, 26*4-15?/= 168. We will now show another operation to reduce this kind of equations. Take the lowest coefficient, (in this example it is 15,) and observe whether it will divide the other numbers or not. If INDETERMINATE EQUATIONS. yi it will divide, reserve the whole numbers, and omit the frac- tions. In this case, 15 will not divide 26, and will divide 168. The quotient will be 11, disregard the remainder. Now assume p=x-\-y — 11. Then, since x and y are whole numbers, p must be a whole number. Multiply by 15, and transpose, and we have 15j9 — 15a; — 15y= — 165. 2llr+15?/= 168. By addition, 15;>{-lla? =3. Assume q=x-]-p. Then - - - - l\q— Up— Ux=0. Sum 11 cy' r\ ■ ^ cx ' A -^ C V' Or, m<—-. and m > — ^ b ' ^ a That is m at the same time is found to be greater and less than known quantities, therefore its limit or range is found. For instance, if m must be greater than 30, and less than 40, we conclude that it may be any number between 30 and 40, and the number o£ different values it can take is 9. We perceive that the difference between the integral parts of — — and — will express the range of m, and the number of different solutions which the equation admits of, (except in certain cases ; ) as m is more than one of these fractions and less than the other, the difference between the expressions — and — is sometimes one more than the number of dif- o a ex ferent values of m, such is the case when — is an integer, in such cases, subtract one from the difference of these quan- tities for the range of m, but this case very seldom occurs. INDETERMINATE EQUATIONS. 97 N. B. In making use of the expression — and — care ' Cb must be taken, not to take their difference as factional ex- pressions, their absolute difference is not wanted, it is the ex' cii f difference between the integral farts of — and- 1 -. ° r b a Example. Required the number of integral solutions to the equation 7*4-%= 100. Find the least value to a?', y' in the equation^' — 9/=l. lx' — 9y'=l. Assume p— x' — y' Then - - lp— 7x'+7y'=0. Add, and lp — 2y'=l. Assume q=3p — y' Then - - 2q— 6p+2i/=0. Add, and 2q-\- p=l. Or, p=l — 2q. Take £=0, then p=l. y'—% # =4. mL ex' 100-4 j cy 100-3 Then — =— — - and -£ = — — . b 9 a 7 That is ^=441 and ^-=42« b 9 a 1 Disregarding the fractions, the difference of the integral parts is 2, that is there are two integral solutions to the equation. 400 300 9 7 .... 2800 2700 100 13 _ fractional form, thus: — ^ .— - =—=137. Here the integral difference is one, which without this caution might be taken for the number of solutions. The integral difference in this case is not the difference of the integrals. Observe in this example 44£ and 42|, the fractional part of — is |, and the fractional part of — is f , the former is o a less than the latter* in such cases the integral parts must be taken separately. If we had taken the difference between — — , in a 98 KEY TO ALGEBRA. ex But when the fractional part of — - is not less than the b cv fractional part of -— , but equal or greater than it, we may- find the number of solutions by taking the difference of the ex' cy' _ , expressions — . Reduce to a common denominator, and take the difference of the numerators, and we will have — - — — ; but ax' — by'= 1. Therefore, we have — ab ab for the number of solutions, at once. Example. What number of integral solutions will the equation 1 9,z-f-l 3j/ =2000 admit of? Ans. 17. ~9 X 13=117) 2000(17. But as we cannot know whether the fractional part of C11 CI J ~y~ is not less than the fractional part of — we cannot be b r a sure that dividing c by ab will give the true number of solu- tions. It either will be the true number or one less. The equation 5#-|-9#=40 admits of no solution in whole numbers, c=40 will not be divided by aZ>=45. Now take the equation hx' — 9#'=1. ' And we find x'=2, and y'=l. ex' 80 cy' 40 Therefore, — = n =8£, and — = - =8. Now as there b 9 y a b is no difference between these integral parts, it indicates as it should, that there is no solution. But let us take 5#-f-9//=37, the same equation, except a smaller value of c. If c would not divide by ab before, much less will it now. Yet in this last equation we have ex' 74 a solution, a?' =2, y'=l, as before, and c=37. — =-^-=8|- and — = _ =7|. Here the difference of the integrals is 1, a 5 and indicates one solution. x=2, y=3. INDETERMINATE EQUATIONS. 99 How many solutions will the equation 2a:-)-5^=40 admit of? The auxiliary equation 2a?' — 5/=l, gives #'=3, y'=l. «*[_. 24. ^=20. Or, 4 solutions. C2?' But observe that ~r in this case, is a complete integral, 24 ; agreeable then, to previous considerations we must de- duct one, and the number of solutions are but 3, as follows: #=5. 10. 15. #=6. 4. 2, and no other solution can be found. What number of solutions. in whole numbers can be found for the equation 3x-\-5y-\-lz=\00. As x and y each cannot be less than one, z cannot be greater than = =131. That is, z cannot be greater than 13, in whole numbers. Now suppose 2=1, and the equation becomes 3x-\-oy=93. The number of solutions for this equation, found as pre- viously directed is 6. rm, of - J a? = 26. 21. 16. 11. 6. 1. inatis^ ?/== 3 6 9 ]2 15 1Q Now x and y can make these 6 changes, and z be con- stantly equal to 1, and satisfy the primitive equation. Take z=2, and the equation becomes 3#-j-5 < y=86. This equation has also 6 solutions, z being through all the changes of x and y equal to 2. Now take z=3, then the original equation is 3z-|-5y=79. This equation has five solutions. Now take z=4, then 3.r4-5y=72. This equation has four solutions. Take z=5, then 3#-|-5y=65. This equation has four solutions. Take 2=6, then 3z-[-5y= 58. This equation has four solutions. In this manner, by taking z equal to all the integers up to 12 in succession, we find 41 solutions. 100 KEY TO ALGEbKA. THE DIOPHANTINE ANALYSIS. The Diophantine Analysis teaches hew to find square and cube numbers under given conditions, or having given relations to each other. Examples. Case 1st. Find such a value of x as will make the ex- pression ax-\-b a square. Put ax-\-b=n 2 , n 2 being any square, it is, therefore, an indefinite problem. ' _ . . n 2 — b . - From the equation x= ; take n equal to any num- ber whatever, and a and b being known, x becomes known. Eight times a certain number added to 9, makes a square. What is the number ? Let a? = the number. Then Sx-\-9=n 2 , that is any o i n a n2 — b n2 ~ 9 A square. a=o, o=9, and x= = — — — . Assume n a o =7, then #=5, the required number. But there are many other numbers that will answer the condition according as we assume n more or less. Find x, such that the following expressions shall be square numbers. 9*4-9. 7#+2. 3x— 5. a?+|. All these correspond to the general expression ax-\-b. Case 2d. Any algebraic expression in the general form of ax 2 -\-bx, may be made a square by supposing its square root equal px; x must be in some part of the root, because the expression contains a: 2 , that is some function of a?. Now if px is the root, ax 2 -\-bx=p 2 x 2 . Divide by*, &q and we have %=--£ — • p may be any assumed value whose square is greater than a, DIOPHANTINE ANALYSIS. 101 Examples. Six times the square of a certain number, added to five times the number is a square. What is the number ? 5 6x 2 -\-5x=p 2 x 2 . Or, x= -• p 2 — 6 Here it is obvious that p 2 must be greater than 6, other- wise it is unlimited. Take p=3, then a?= f = If. x 2 Find the value of x to make — -\-3x a square. Ans. #=6. Case 3d. Any algebraic expression in the general form of x 2 ±bx-\-c, can be made a square, by putting xdzp equal its square root. We can if we please take x — p for the root in all such cases. Then if p is less than x, the square is diminished, if greater, the whole root will be essentially minus, but the square will be plus, and may rise to any amount. Therefore x — p is far more general than x-\-p. Case 4th. Any expression in the form of ax 2 dzbx-\-c 2 , can be made a square, by taking its root equal to cdzpx. It will be observed that x must be in the root of the pre- vious expression, because it has x 2 , and c must be in the root of this last expression, because it contains c 2 . In the first we have x 2 ±bx-\-c=x 2 ±2px-{-p 2 . Or, p 2 — c In the second, x 2 ±bx-\- c 2 —c 2 ±: 2cpx-{-p 2 x 2 . Or, x±:b= ±3cp-\-p 2 x. Or, x= - ^~- In both cases as- Idap 2 sume p of any convenient value to render x positive, and as small as possible. Find a number such, that if it be increased by 2 and 5 separately, the product of the sums shall be a square. Let z=the number, then (a?4-2) (#+5) =a? 2 +73+10, 102 KEY TO ALGEBRA. must be a square. b=l, c=10. General solution, x=J—^ — 2p±7 Now p 2 must be more than 10 ; hence, take j?=4, and *= T 6 5 = |, the least number that will answer the conditions. Case 5th. An expression in the form of ax 2 -\-bx -\-c, where neither the first nor the last terms of the expression are squares, neither branch of the root can be directly found, and the expression cannot be made a square, unless we can separate it into two rational factors, or unless we can first subtract from it some simple binomial square, and can then divide the remainder into two rational factors. By reminding one of the nature of quadratic equations, all may perceive that the expression ax 2 -\-bx-\-c must be the product of two factors, but whether rational factors or not is the subject of inquiry. To find the factors which make the product ax 2 -\-bx-\-c, put this expression equal to 0, and work out the values of x thus, ax 2 -\-bx-\-c=0. Or, ax 2 -\-bx= — c. Complete the square, and 4a 2 x 2 -\-\abx-\-b 2 = b 2 — \ac. Or, 2ax-\-b= ±J \b 2 —4ac.\ We now perceive that the values of x must be rational, provided J \b 2 — 4ac\ is a complete square. If it be so, let la-J \^-iac\-l=n h and -I J («•-*«}- £•* Then the two values of x are x = m and x=n, and (a? — m) (x — n), are the factors which will give the expression ax 2 -\-bx-^-c. Examples. 1. Find such a value of x as will make 6a? 2 -|-13x-|-6 a square. Here «=6, £=13, e=6. £ 2 =169, 4ac=144, Z> 2 — lac =s25 and J \ b 2 —4ac } =5. Now 12o:+l3=±5. *=--§ , Or, *=— |. Or, 3z+2=0, and 2*4-3=0. DIOPHANTINE ANALYSIS. 102 That is, (3x-\-2) (2#-|-3) will produce the expression 6x 2 +13tf+6. Now to make the expression a square, put (3*+2) {2x+3)=p 2 {3x+2) 2 Then 2x+3=p 2 (3z+2.) And »=-?£_. Take p=l, and x=l. We might have seen at first, that in this particular expression, the value of x being 1, would make it a square, as 6-j-13-|-6=25, a square. That is in all cases when the sum of the coefficients make a square number, the value of x may be one. 2. Find such a value of x as shall render the expression 13x 2 -\-]5x-\-7*a. square. Here as neither the first nor last terms are squares, nor b 2 — 4^ a square, the expression cannot be made a square, unless we can separate the remainder into factors after taking away some simple square. But in this case, \ac is greater than b 2 , we must then, in subtracting our square, diminish a and c and increase b. To accomplish this end we will subtract the square of * — 1, not x-\-\. That is 13z 2 +15z+7. Subtract x 2 — 2.r+l. Remainder,- - - - l2x 2 -\-\lx-\-Q. In this last expression, a=12, £=17, cz=6. Hence, b 2 — 4ac=289 — 288=1, a square ; we are now sure ra- tional factors can be found to produce the expression 12z 2 +17a?-|-6. By assuming 12 2 -}-17a;-|-6=0, and finding the values of s by the quadratic, (merely to get the factors,) we find «ass— |, and a?== — |. Or, 3; +2=0, and 4*+3=0. And (3z+2)(4z+3)=12* 2 +17z+6.* ♦The values of x used to obtain these factors have no connection with tat values of z to render the original expression a square. 104: KEY T0 ALGEBRA. Now the original expression is the same as (a? — l) 2 -f» (3a?-f 2) (4a?+3.) Put its root equal to (x— l)+^(3a?+ 2 )) and square this root, and (a?— l) 2 +(3a?+2) (4a?-f-3)=(a?— 1) 2 + 2^(a?— 1) (3a?+ 2)+ /(3a?+2) 2 . By reduction, 4a?+3 = 2p{x—l)+f[3x+2. Or, (— 4+2p+3/)a?=2p+3— 2p 2 . Therefore *- 2 ^+ 3 — 2 ^ inerelore, ^^—-^ Take^=l, then a;=3, and 13:r-J-15a?-}-7=169a square, Ryan makes a; in this example equal i and the expression equal *§ l a square, but an integer number is always more satisfactory. 3. Find such a value of a? as will render 14a? 2 -j-5a? — 39, a square. After a few trials this expression is found to be the same as (2a? — l) 2 -|-(5a? — 8)(2a?-|-5.) Assuming its root to be 2a? — l-\-p(5x — 8 ) Then by squaring the root, making it equal to its power, and reducing, we find a? = , 7T A < / Assuming p=l, a?= 1 T 5 , and the expression equals 36, a square. Other values can be found, by assuming different values to p. 4. Find such a value of x as shall make 2a?*-(-21a?-|-28 a square. After a little inspection we find this expression equal to (a;+4) 2 +(a?+l) (a?+12.) Now if we make (z+4) 2 +(*+l) (a?+12)= {(*f4>-p(*+l.)J» After reduction, we shall find a?= , ^ 1 • pr — 2/? — 1 Assume p=4, then a?=4, and the original expression is 144 a square. If (a?+ 4) 2 +(a?+l)(a?+12)={(a?+4)-^(a?+12)} 2 , we shall find a?= ^~ P ~ . If we take^=l, a?=f . If we DIOPHANTINE ANALYSIS. 105 take 2?=!, a?=8, and we might find many other numbers that would answer the conditions of the expression. Case 6th. When we have an expression in the form of a^x^-^-bx^-^-cx'-^-dx-j-e, we can assign a value to x that will make the whole expression a square, if we can extract three terms of its root. Assume such terms as the whole root, square the root so assumed, making it equal to the given expression, and by reducing, we shall have a value of x which will make the original expression a square. Example. Find such a value of x as shall make 4x 4 -\-4x 3 -\-4x 2 -\- 2x — 6 a square. We commence by extracting the square root as far as three terms, and find them to be 2# 2 -|-#-|-£. Now assume 4z +4x 3 -\-4x-+2x— 6=(2z 2 +#+!.) 2 3a? 9 Expanding and reducing, we have 2x — 6=— — |-— . And x=\3\. Essentially the same method must be pursued in other problems of the like kind. Case 7th. Find such a value of x as shall render 2ar 2 -f-2 a square. Expressions of this kind, when neither a nor c are squares, nor b 2 — \ac a square, and which cannot be resolved into factors, presents an impossible case, unless we can first find by inspection, some simple value of x that will answer the condition. In the present example, it is obvious that if #=1, the expression is a square. But we wish to find other values than 1 that will render this expression a square, and having found that one will answer, we are now enabled to find other val- ues, thus : Let a?= l±y. Then x 2 =lzk2y+tf. 106 KEY TO ALGEBRA. And 2x 2 -|-2=4d=%-{-2y z . Here the original expression is transformed into another expression, having a square for its first t^rm. Now we must find such a value of y as shall make 4-j-4?/-|-2y 2 a square. Assume 4-}-4^-|-2^ 2 = (2 — my) 2 — 4 — 4;my-\-m 2 y z . Or, a i r> /I i \ T1 4(m-f-l) 4+~j/= — lm-\-m-y. Hence #=s -^f- 9 i m mav De an y number greater than one. Put m=2. Then #=6, and 5c=l-(— y=7, and the original expression 2;r 2 -f-2=98-}-2= 100, a square. N. B. It often occurs incidentally in the solution of prob- lems, that we must make a square of two other squares. This can be done thus : Let it be required to make x 2 -\-y 2 a square. Assume x=p 2 — q 2 , and i/=2pq. Then, x 2 =p* — 2p 2 q 2 -\-q\ And, y' 2 = 4p*q 3 . Add, and - - x 2 -\-y-=p -\-~p 2 q 2 -{-q 4 , which is evidently a square, whatever be the values of p and q. We can, therefore, assume p and q at pleasure, provided p be greater than q. Double and Triple Equalities. In the preceding section it was only necessary to find such a value of the unknown term as to render a single expres- sion a square. But there are problems where it becomes requisite to find such a value of the unknown term as to render several different expressions squares at the same time. And this is called double and triple equalities. Case 1st. As a general equation for double equality, let it be required to find such a value of x that ax-\-b may be D10PHANTINE ANALYSIS. 107 a square, and the same value of x give cx-\-d a square. f2 Q Let ax-{-b=t 2 . Then x. And cx-^-d=p' 1 . Then x= a p 2 — d Therefore = — . Or, ct 2 — cb = ap 2 — ad. a c $ Tranpose cb and multiply by c and we have c 2 t 2 -=acp 2 -\-c 2 d — acd. As the left hand side of this equation is a square, what- ever may be the values of c and t, it is now only requisite to find such a value of p 2 as shall render the other side a square, which can be done by some one of the artifices in the preceding section. To illustrate this we give the following definite problem. The double of a certain number increased by 4, makes a square, and five times the same number plus one, also makes a square. What is the number * Let x represent the number. Then 2^+4= * 2 ] J> From which < And 5a?-j-l=p 2 x= — 5 Hence 5* 2 — 20=2p 2 —2. Or, 5* 2 =2j9 2 +18, multiply by 5, and 25^ = 10;? 2 +90. The left hand side of this last equation being a square, whatever be the value of t, it is now only necessary to find such a value of p 2 as to make 10^ 2 -f-90 a square, an expression which corresponds to case 7 of the last section. We therefore cannot proceed unless we find by trial, by observation, by intuition as it were, some simple value of p that will make 10/> 2 -|-90 a square, and we do perceive that if p= 1, the expression will become 100, a square. Now if ;?=1, it will give a definite and positive value to *, and the problem is solved. If not we must find other values of p. 108 KEY TO ALGEBRA. But we have found that #= — - — , and if p=l, £=0, and the original expressions 2z-|-4, and 5#-|-l, become 4 and 1, squares it is true, which answer the technicalities, but not the spirit of the question. ^To find another value of p. Put p = 1 -|- q. Then 10^ 2 +90 = 100*-f20;?+2 ? 2 . To make this a square assume 100-\-%0q+2q 2 = {l0— nq) 2 = 100— %0nq-\-n 2 q 2 . By reduction, q= - —}— •". . Now n must be so taken, J a n 2 — 10 7 that n 2 is more than 10: take ti=5 and ^=8, ^=9, then #=16 and the original expressions 2#-f- 4 = 36, a square, and 5#-|-l=81j a square. Case 2d. A double equality in the form of ax 2 -\-bx=Q and cx 2 -\-dx, also equals a square, may be resolved by making *=— , tlten the expressions will become — (a-\-by) and — - (c-\-dy,) which must be made squares. But if we multiply a square by a square, or divide a square by a square, the product or quotient will be square. Now as each of the preceding expressions are to be squares, and as they obviously have a square factor — it is only necessary to make a-\-by, and c-^-dy squares, as in the first case. We may also take another course and assume ax 2 -\-bx =p 2 x 2 , which gives x= - , which value put in the other expression, and we have c (^^) +\^ZZ^) ==D ' Multiplying this by the square (p 2 — a) 2 , and the expres- sion becomes cb 2 — dbd-\-abp 2 = some square, from which the value of p can be found and afterwards x. y And DIOPHANTINE ANALYSIS. IQg Example. A certain number added to its square, the sum is a square, and the number subtracted from its square, the remainder is a square. What is the number? Let x = the number. Then x 2 -{-x=Q. And x 2 — #== some other square. Assume x=— • Then — ( \-\-y ) = Q. The problem will be solved if we can find such a value of y, as will at the same time make \-\-y and 1 — y squares. Therefore put l-^-y=p 2 , and 1 — y=q 2 - From the first, y=p 2 — 1. And y=l — q 2 . Therefore, q 2 =2 — p 2 . As q 2 is a square, we have only to find such a value of p 2 , as shall render 2 — p 2 a square. But this cannot be done unless we can find some simple value of p by inspection, and we do observe it must be one. But p being equal to one, gives y=0, which will not answer the conditions. Therefore, let p=l-\-t. Then 2— p 2 = \— 2t— t 2 ={\— ut) 2 = l— 2ut+u 2 t*. 0r > fa ^pT' Take«=* <_} p=l+t=\. y=p 2 — 1=| j- x= — • x = ||, a number that will an- swer the given conditions. Case 3d. To resolve a triple equality. Equations in the form of ax -\- by = t 2 , ax-\-dy=* u*i ez-{-fy=s 2 , can be resolved thus : ■r> • c j dt 2 — ^ 2 By expunging ?/, we find x= — j — - — ^2^,2 ct 2 Then by expunging x. y=-~- — — - . Substituting these values of a; end y in the tnird equation 110 KEY TO ALGEBRA. j i. ni. ( a f— bc)u 2 +(de— cf)t 2 , and we shall have ^ '- — P~\ J — L — =s 2 . ad — be Assume «=±te. Then u-=t 2 z 2 . Put this value of u 2 in the above, and divide by f, and we shall have (af — bc)z 2 -\-de — cf__s 2 ad — be ~~ t*' The right hand side of this equation is a square, and therefore all that is now requisite, is to find such a value of z as shall make the other side a square, which when possi- ble, can be done by case 7, section 20. After z is found t may be assumed of any convenient value whatever. Now u is known, and with t and u known quantities, we know x and y. The preceding are some of the most comprehensive and general methods yet known ; but there are cases in practice where no general rules will be so effectual, as the operator's own judgment and penetration. Much, very much will depend on skill and foresight dis- played at the commencement of a problem, by assuming convenient expressions to satisfy one or two conditions at once, and the remaining conditions can be satisfied by som.6 one of the preceding rules. Examples. 1. It is required to find three numbers in arithmetical progression, such, that the sum of every two of them may be a square. Let x 1 x-\-y and x-\-2y represent the numbers. Then by the general formula, 2x+y=t 2 , 2x-{-2y=u 2 , 2x-{-3i/=s 2 . -n • • i t* — V u2 — % By exterminating x, we have ■ — ^-'-= — jz — -• Continuing thus after the general equations, we* find a DIOPHANTINE ANALYSIS. HJ long and troublesome process, and in conclusion, we find the numbers to be 482, 3362, and 6242. The above is according to the common as well as the general method. The following is Mr. Young 1 s Solution. Let x — y, a?, and x-\-y represent the numbers. Then 2x—y, 2a?, and 2x-\-y must be squares. Assume 2x=m 2 -\-n 2 , and y=2mn. Then 2x — x=m 2 — 2mn-\~n 2 , and 2x-\-y=m 2 -\-2mn-\-n* are evidently squares. It therefore only remains to make 2x or m 2 -\-n 2 a square, and this can be done as explained at the close of the last section by assuming m=r 2 — 5 2 , and n=2rs. Then 2x=.m 2 -\-n 2 =^{r 2 -\-s 2 Y^ an expression in which r and s can be assumed in numbers. But they must be so assumed that x shall be greater than y to make x — y the first number, positive and for this reason, we must give the literal expressions for the numbers before taking definite values for r and s. The expressions for the numbers are S&— y= £(r 2 +s 2 ) 2 — 4rs (r 2 — s 2 .) x= £(r 2 -f-5 2 .) 2 x-\-y— ±(r 2 -\-s 2 ) 2 -\-4rs(r 2 =s 2 .) Take r=9, 5=1, and 482, 3362, 6242, are the numbers. Another Solution. x 2 x 2 x 2 Let -^ y->~n an( ^ o — |-y be the numbers. Then x 2 — y, x 2 -\-y, and x 2 must be squares. But the last being a square, we have only to make x 2 — y and x 2 -\-y, squares. Assume y=2x — 1. Then x 2 — y=x 2 — 2#-|-l, a square, and we now have only to make x 2 -\-2x — 1, a square. Therefore make * 2 -l-2# — \=(x-\-nf—x 2 -\-2nx-\-n 2 . X -2{l-n) It is manifest that n must be less than one, make it .& „, 1.64 41 _ x 2 1681 72 1440 Then ... -=- Or, - = — ^fo^W 112 KEY TO ALGEBRA. m , 482 3362 6242 Then 400' 400 ' 400" are the numbers. Here we have the numbers expressed in fractions, but the denominators are common, and is a square number, we may therefore multiply all three by 400, and we shall have 482, 3362, and 6242 for the numbers, as in the other solu- tions. If we take n= f in this last result, we shall have 2162, 7442, and 9442 for the numbers. 2. Find two numbers such, that if to each, as also to their sum, a given square, a 2 be added, the three sums shall all be squares. Let x 2 — a 2 and y 2 — a 2 represent the numbers : then the first conditions are satisfied. It now remains to make x 2 -\-y 2 — 2a 2 -\-a 2 a square, or, ff'+y 2 — & 2 =n. Assume y 2 — a 2 =2ax-\-a 2 . This assump- tion will make the expression a square, whatever be the values of either x or a. But the assumed equation gives y*=2&a+2& 2 , and as y 2 is a square, we must find such values of x and a, as shall make 2ax-\-2a 2 : a square. Put x=na. Then 2na?-\-2a 2 = □ ,or, a 2 (2?i+2)= □ . Hence it is sufficient that we put 2^+2 = some square. Therefore, assume 2?i+2=16. Hence n=7 and x=7a. Now take a equal to any number whatever. If a square. ©2 1 Therefore, put y 2 -^-\z=(y — py y which gives y—~ 5— i ' tip Take ^=1, then #=£, and y 2 -\- |=§f=# 2 . Therefore, oVj If? if? are tne numbers; but we can multiply them all by the same square number 64, and their arithmetical rela- tion will not be changed, and they will still be squares ; hence 1, 25, and 49 may be the numbers, or 4, 100, and 196, 5. Find two whole numbers, such that the sum and dif- ference of their squares, when diminished by unity, shall be a square. Let #-f-l=one number, and y= the other. Then by the conditions we must make squares of x 2 -\-y 2 -* r 2x, and K 2 — ^2_|_2#. Assume 2x=a 2 , and y 2 =2ax, then the ex- pressions become x 2 -\-2ax-\-a 2 , and x 2 — 2ax-\-a 2 , obvious squares, whatever be the values of x and a. But the equa- tions 2x=a 2 and y 2 =2ax must be satisfied. Take a=4, then a?=8, y=S, and #-(-1=9. Therefore 9 and 8 are the numbers. 6. Find three whole numbers, such, that if to the square of each, the product of the other two be added, the three sums shall be squares. Let z, xy, xv : be the numbers. Then by the conditions, x 2 -\-x 2 yv, x 2 y 2 -\-x 2 v, x 2 v 2 -\-z 2 y, must be squares. As each term contains a square factor # 2 , it will be sufficient to make l-{-yv=n 1 y 2 -\-v=D 1 and v 2 -\-y=n> Assume i/=4v-j-4, and this will make the first and last expressions squares. Substitute this value of y in the second expression, and we shall have 16v 2 -j-33r-|-16, which must be made a square. Hence put 16^ 2 -)-33v-|-16=(4 — fv) 2 , which reduced gives v= 2 J L Take ^=5, then v= 7 ^ Now take #=9, and we have 9, 73, and 328 for the numbers. 7. Find two whole numbers whose sum shall be an inte 8 H4: KEY TO ALGEBRA. gral cube, and the sum of their squares increased by thrice their sum shall be an integral square. Let x-\-y=n 3 , that is some cube. Then x 2 -\-y 2 -\-3n 3 =z □ . Put 2xy=3n 3 , then x 2 -\-2xy-\-y 2 is a square, whatever may be the values of x and y. But x and y must conform to the equations x-{-y=n 3 , and 2xy=3n 3 . Work out the value of x from these equations, on the supposition that n is known, and we shall find 2x=?i 3 -\- t J \n 6 — 6ft 3 .} Now x will be rational, provided we can find such a value of n as shall render n 6 — 6n 3 a square, but if we add 9 to this, we perceive it must be a square, and we have two squares, which differ by 9. Therefore one must be 16, the other 25, as these are the only two integral squares which differ by 9. Hence n 6 — 6?i 3 +9 = 25. Or, ?i 3 —- 3 == 5. n 3 =8, 7i=2, and #==6, y=2. 8. Find three numbers, such that their sums, and also the sum of every two of them, may all be squares. Let x 2 — 4a; = the first, 4x= second, and 2#-|-l= third. By this notation, all the conditions will be satisfied, except the sum of the last two. That is Qx-\-l must be a square, but fro have three differe?it whole numbers, no square will answer under 121, the square of 11. Hence put 6x-\-l = l21. Or, a?=20. And the numbers will be 320, 80, and 41. 9. Find two numbers, such that their difference may be equal to the difference of their squares, and the sum of their squares shall be a square number. Let x and y be the numbers. Then x — y=x 2 — y 2 . Divide by # — y, and l — x-\-y. Hence a? = l — y, and z 2 -f-y 2 =l — 2j/-f-2y 2 . Which last expression 1 — 2y-\-2y 2 must be made a square. For this purpose, put 1— %+2# 2 =(l— nyf. Hence y=- J|^ • Take n any value to render y less than one in order to DIOPHANTINE ANALYSIS. ^5 give x a positive value. Therefore take w=3, and y=z 4 Consequently x= f , answer. 10. Find three numbers in geometrical progression, such that if the mean be added to each of the extremes, the sums in both cases shall be squares. Ans. 5, 20, and 80. 11. Find three numbers, such, that their product increased by unity shall be a square, also the product of any two in- creased by unity, shall be a square. Ans. 1, 3, and 8. Assume 1 for the first number, and x and y for the other 12. Find two numbers, such that if the square of each bo added to their product, the sums shall be both squares. Ans. 9 and 16. 13. Find three integral square numbers in harmonica! proportion. Ans. 25, 49, and 1235. 14. Find two numbers in the proportion of 8 to 15, and such that the sum of their squares shall be a square number. Ans. 136 and 255. Bonnycastle's answer, 476 and 1080. 15. Find two numbers such that if each of them be added to their product, the sums shall be both square. , Ans. i and J. The above require no explanation from us. There are many severe and tedious problems in the Diophan- tine Analysis, proposed by Bonnycastle, Young, and others, which require more time and practice than algebraists in general ought to give for the advantage derived, as time and thought may be better employed in Analytical Geometry, the Calculus, or Astronomy. 116 KEY TO ALGEBRA. THE DIOPHANTINE ANALYSIS. The Diophantine Analysis is sometimes useful in solving numerical Equations, in which squares and cubes are in- volved, as the following examples will show. 1. Given j ^__ _ 7 \ to ** n( * one va * ue °^ x an< * V' By subtraction, and the transposition of y 2 we have x 2 +x-\-y=y 2 (a) As the second member of this equation is a square, the first must be a square in fact, if not in form. But, we perceive, that if we put y=x-\-\, in the first member, it will be in a square form. Put this value of y, in the first equation, and we have x*-\-x=6; which gives #=2; hence y=3; values which verify both equations. N. B. This method of operation is not general. It only serves to resolve particular cases. We might have made the first member of equation (a), a square, by putting y—3x-\-4, or 5#-f-9; but the results of these substitutions would not verify the primitive equations. 2. Given \ 2x *— 3x y+ 2/ 2==4 ? to find values of x and v. \ # 2 +3y 2 — 2xy=9 ) As 4 and 9 are squares, the first members are square in fact, though not in form. But we can make the first mem- bers square in form, by assuming 2x 2 —3xy =0, and 3y 2 — 2#i/=0. Then y 2 =4 and # 2 =9, or i/=2 and #— 3 ; va- lues which verify all the equations. DIOPHANTINE ANALYSIS. 117 3. Find such integral values of x, y, and z, as wiil verify the equations .... x i -\-y iJ txy=31 i and . . x 2 -)rz i -{-xz =49. If we add xy to the first equation, and xz to the second* the first members will be square ; and, of course, the second members will be square in fact, though not in form. We have, then, to make 37-f-#y> and 4§-{-xz t squares, *o accomplish this. Put 37-ftft/=49, or xy=12 (1) and 49~\-xz=64, or xz=\5 (2) 12 15 From (1), • . a?= — ; from (2) . . x=— . y * Hence « • 12z=15y, or, z=— . Take y=4, then z=5, and x=3 ; values which will verify the given equations. 4. Find such integral values of y and z as shall verify the equation .... *y 2 -]-z 2 -\-yz=6l. Add yz to both members, then put 6l4-y*=n2. Now if we assume n=8, yz=3. But yz—3 will give y-\-z=8, and these two equations will not give integral values to y and z. Therefore, take n=9, then w 2 =81, t/z=20, and y-f-z=9. Hence z=4 or 5, and y=5 or 4. 5. Find such values of x and y as will verify the equa- tions xy-\-xy 2 =l2 . (1) and. ... x +xy*=\8 (2) 12 Equation (1) may be put into this form y*— — — y. . .(3) * # 18 Equation (2) into this y 8 = 1...(4) x The first member of equation (4) is a cube ; therefore Put 11-13=8. Whence z=2. x 118 KEY TO ALGEBRA. r - y-\-u-{-y 2 u 2 =^2l [to find one value of each | x-\-v-\-2?v 2 =41 | of the symbols. A regular solution would result in a very high and tedi- ous equation ; but if the values are integral, we can soon determine them as follows : Take the first equation, and put v-\-u=s, and transpose s. Then vhi 2 =l3 — s; which shows that 13 — s must be some square; and if s is positive, the square cannot be greater than 9. That is, 13 — #=9 or, s=4. Then v-f-w=4, and vu=3 ; giving v=l or 3, and t/=3 or 1. By taking u=l 9 in the second equation, we find y=4. With the values already found we obtain x, from either the third or fourth equations, v=3, w=l, a?=2, y=4. 7. Given \ ^~~~ ( to find values of x and y. { 2?/ 2 -f-3.ri/= 8 J * Put xy=p ; transpose, &c, we have 4# 2 =12-{-2;) and 4i/ 2 =16— 6p. Now we must find such a value of p as shall render the second members of these last equations square at the same time; which is p=2; this gives 4x*= 16, x=2. ft p. C 6x 2 -\-2y 2 =5xy -f- 12 ) to find one value of ( 3x 2 -\-2xy=3y z — 3 ) x and y. This problem is under (Art. 110) of both editions. Ac(^ the equations together, and reduce, and we have 9x 2 =y 2 -\-3xy-{-9. The first member of this equation is a square ; therefore the second member is a square, but to make it a square inform, as well as in fact, we perceive it is only necessary to make MISCELLANEOUS EXAMPLES. H9 07=2. Or call y 2 -\-3xy-\-9 a binomial square, and decide the value of x agreeable to section 8, This gives a?=2, the answer. q p- 5 ^ 2 — *'/= 28? to find the rational values ' ° 1V ^4y 2 4-3ary=160 5 of a? and y. (Y. 139.) Ans. x = ±4, y= ±5. There are, and may be equations of the preceding forms which have no rational values, such are not susceptible of of this mode of treatment. OF THE \ Section XXIIli UNI VERS IT Miscellaneous Exampl %IP0R^' 1. When wheat was 8 shillings a bushel and rye 5, a man wished to fill his sack far the money he had in his purse. Now if he bought 15 bushels of wheat and laid out the rest of his money in rye, he would want 3 bushels to fill the sack: but if he bought 15 bushels of rye, and then filled his sack with wheat, he would have 15 shillings left. How much of each must he purchase to fill his sack, and lay out all his money ? (Colburn, page 50.) Ans. 10 bushels of each. Solution by Mr. T. J. Matthews. Let x= the wheat, and y= the rye. Then it is evident that when he buys 15 bushels of wheat he has too much, as he has not money enough left to fill his sack with rye. Now 15 — x is the excess of the wheat purchased above what he ought to have had, and this excess of quantity, multiplied by the excess of a bushel of wheat above one of rye, wiJJ give the deficiency of his money, or equal to 3 bushels of rye at 5 shillings. Consequently, 3(15 — a?) =15, or #=10. By similar reasoning, it will appear that when 15 bushels of rye are purchased, he buys too much, and the excess is 15 — y, which multiplied as before, will equal the excess of 120 KEY TO ALGEBRA. his money, viz: 15 shillings. Therefore 3(15 — y)=15, and y= 10. 2. A person bought two cubical stacks of hay, for ^41 ; each of which cost as many shillings per solid yard as there were yards in a side of the other, and the greater stood on more ground than the less by 9 square yards. What was the price of each 1 (Colburn.) Solution by T. J. Matthews. Assume 5z, and 4z the sides of the Cubes. Then 25a; 3 — 16^=9, by the first condition. Therefore, x=h 1254=500. 64-5=320. 3 Or, £26. £IQ. Answer. 3. Given x-\-y-{~z=25. ary=6 #2=60, to find #, y and z. Solution. From the two latter, z—lOy. Then the first becomes x-\-lly—25. Or z 2 -\-Uxyz=2ox, but from the 2d, ll2?y=66. Hence ^—-25^=— 66. (A) Assume 2«= — 25, then 6&-f-9= — 66, and equation (A) becomes (z 2 -{-2ax-{-d i =a 2 -{-6a-{-9. Or, a?=3,) 4. Given a^=125#-|-300y. And y 2 — x 2 =90000, to find * and y. (Young p. 146.) Ans. #=400. y=500. Put y =px, then px 2 = 1 25#+300^#. ( 1 ) And p 2 y 2 —x 2 =(300) 2 (2) 125 From equation (1) p= ~~^ (3) (300) 2 From equation (2) p 2 — l = v — — THe right hand side of this last equation being a square the other side is also a square, and one accustomed to the analysis will perceive that p must equal f, to make the ex- pression p* — 1 = D- Others can go through the form and they will find that p=l, which value put in equation (3) gives a?a=400. MISCELLANEOUS EXAMPLES. 121 It is not imperative that we should resort to the Diaphan- tine to solve this problem but it is very Convenient. 5. Find two numbers, such that the fifth power of one may be to the cube of the other, as 972 to 125. Ans. 6 and 10. Let a? and nx be the numbers. Then a; 5 : n 3 x 3 : : 972 : 125. Or, x 2 : n 3 : : 972 : 125. Multiply the first and third terms by x, x 3 : n 3 : : 972a: : 125. 1 25x 3 Therefore, 972*=— f-. n 3 The right hand side of this equation is a cube, therefore, 972#= a cube: Or, 27*36x=a cube. Hence, 36a? must be a cube, which it evidently is, when #=6, as 36 is the square of 6. 6. Given x +i,+xy(x+y)+r* y 2 =85 ) fi , , And xy+{x+?jy+xy(x-\-y)=97 \ V ' Young. 145. Ans. x=Q y=l. Put (x-\-y)=s xy=p, then the equations become s-\-sp+p 2 =85 } And p-\-sp-\-s 2 =97 $ By addition (s-\-p)+s- -\-2sp-\-p 2 =\82. (1) Assume Q,=s-{-/>, then equation (1) becomes Q,2_|_ a==18 o Hence Q= 13. Or, s+p=l3. The re- maining steps are obvious. 12 7. Given l Jx-\-l2= — -=, to find x. 144 Square x-{- 12=^= -£-. Put .r-f-5=y. 144 Then y+7= . Or, ^+7^=141 Put 2&=7. Then 18a-f 81 = 144. Hence y>+2ay+a*=a2+l8a~)-8h y-{-a=a-(-9. Or, — a— 9. 122 KEY TO ALGEBRA. & Given J ( a? +y)(«y+l)=18^- l to find z and o. driven, j ( a?2 _jl y 2> ) ^2 ?/2 _^ 1)=208a , 2y 2 J y Ans. o:=2±V 3 - 0r ?±V 3 - y=7=fc4«/3. Or2d=V3. Solution. Take x-\-y=s. xy-\-l = t and xy=p. Thenst=lSp. (1) And (s 9 —2p)(t 2 —2p)=20Sp 2 . (2) Multiply as indicated and take the value of s't 2 from equation (1) and we have 324/— 2p (s 2 +2 2 )+4/=208/. „ , . **4-* a . st By reduction, p= — ' - — . From equation (1) p=z—^. Therefore, ~*~ =-. Assume s—nt. 00 lo Then this last equation becomes, by a little reduction, n 2 +l » „ '" 1 HO =3- Hencew = 3 - 0r '3- This establishes a relation between x -f- y and #y-|-L Another Solution by Charles E. Matthews. Multiply the original equations as indicated, and x 2 y-\-xy - -\-x-\-y= 1 8xy. x*y 2 -{-x 2 y A ^-x 2 -\-y 2 =2Q8x 2 y 2 . Divide the first by xy, the 2d by x 2 y 2 y then x + y +l +1 =ia And x2 +y*+^+}*= 208 - Assume x-\ — =??i. And y-A — =?j. ^x s ^y Then w+w=18. And m 2 -\-n 2 =212. From which »i and ?i are easily found, afterwards x and y. 9. A square public green is surrounded by a street of uni- form breadth. The side of the square is 3 rods less than 9 times the breadth of the street: and the number of square rods in the street exceeds the number of rods in the perime- ter of the square by 228. What is the area of the square ? (Day 307.) Ans. 576 rods. 10. A man wishes to purchase a certain number of acres MISCELLANEOUS EXAMPLES. 1x3 of land for the money he has at his command. Cleared land is worth 10 dollars per acre; uncleared land is worth $8.-— He finds that if he buys 120 acres of cleared land, and lays out the rest of his money for that which is not cleared, he will not get the quantity of land he wants by 25 acres, but, if he buys 220 acres of uncleared land, and then buys a suf- ficient number of acres of cleared land to make up the num- ber of acres he wants, he will have 4 dollars left. How many acres of each must he buy to have the quantity he wishes, and lay out all his money? (Harney page 203. Ans. 20 acres cleared, 218 uncleared. N. B. Call to mind problem first of this section. In working geometrical problems algebraically much la- bor may be saved by paying attention to the relation oi the given numbers. We give the following as illustrative of these remarks. 1. If the perimiter of a right angled triangle be 720, and the perpendicular falling from the right angle on the hy- pothenuse be 144 ; what are the lengths of the sides ? (Day Alg. p. 305.) Ans. 300, 240 and 180. If we use the identical numbers given 144 and 720, as nine-tenths of our teachers do, they will give large and tedi- ous equations, but if we compare 144 and 720, we shall perceive that one is exactly 5 times the other, and consider- ing the nature of similar triangles, we can work on one of only 144th of the linear dimensions of the first, or a triangle whose perimiter is 5, and perpendicular from the right angle 1. Solution. Let x and y be the two sides, then 5 — x — y will be the hypothenuss. And x*-\-y*=(o — x — y) ? , and xy=5 — x — y. Each member of this last equation expresses the double area of the triangle. Put x-\-y=s. xy=p. 124 KEY TO ALGEBRA. Then $ 2 _2/?=(5— s) 2 =25— 10s+s 2 , and ^=5— *. Or, 105 —2^=25 And 25 +2j5=10 By addit'n 12s =35. _ 35 0r ' 5 =12' But s=x-\-y, the sum of the two sides which, taken from 5, or — >, gives— , for the hypothenuse of the small 25 triangle, hence by 144=300, the hypothenuse of the large triangle. 2. The sum of the two sides of a plane triangle is 1155, the perpendicular drawn from the angle included by these sides to the base, is 300 ; the difference of the segments of the base is 495, what are the length of the three sides ? (Day 305.) Ans. 945, 375, 780. Write the given members in order, thus 300, 495, 1155. Divide them by 15, and their relation is 20, 33, 77. The two latter numbers have a common factor 11, which call a. Put 6=20. Then the three given lines will be b, 3a, and la. Let x= the less side, la — z=the greater side, #=the shorter seg- ment, and g-[-3a—the longer segment of the base. Then p*-\-b*=xx*. (1 And y 2 -\-Qa?j-\-9a 2 -\-b 2 =49a 2 ^Uax+x 2 . (2) Subtract (1) from (2), drop 9a 2 from both sides, and di vide by 2a, and 3y=ab — Ix. (3) We write 2b in place of 40, after dropping 9a 2 . From the square of (3) subtract 9 times, equation (l,)and we have — 9b 2 =a 2 b 3 — 14 abx-\-2bx*. Divide by b, afterwards by 2, recollecting that 6=20, and we have — 90=10& 2 — lax-\-x 2 . 9a 2 Add —j-, to both sides to complete the square. MISCELLANEOUS EXAMPLES. 125 Then?|— 90= ® (a»-40)==®-81 = ^->tax+x>. Extract square root- "9= — x. Or, a?=25. Then 25. 15=375. 3. Divide the number 74 into two such parts that the difference of the square roots of the parts may be 2. Ans. 25 and 49. Let x — 1 , and x-\-\ be the square roots, of the two parts. This problem can also be solved by the Diophantine analysis. 4. Given £*+# 2 =45 and— -) — =£, to be solved by the x y Diophantine analysis. Ans. x—6. y=3. 5 Given a? 2 -f-# 2 =45 and (x-\-y)x=54 to find x and y by the Diophantine analysis. Ans. x=Q. y=3. The two preceding should also be worked by common algebra. 6. A and B traveled on the same road, and at the same rate, from Huntingdon to London. At the 50th mile stone from London, A overtook a drove of Geese, which were proceeding at the rate of 3 miles in 2 hours, he afterwards met a stage wagon, which was moving at the rate of 9 miles in 4 hours. B overtook the same drove of Geese at the 45th mile stone, and met the same stage wagon exactly forty mi- nutes before he came to the 31st mrle stone. Where was B when A reached London 1 Solution. Let #= miles traveled by each per hour, and y= distance B was behind A, then 50 — 2x= the distance from London where A met the wagon. Also,~- J g__ — — time elapsed between the meeting 9y of the wagon with A and B, therefore -.—- ?-x= distance tra- 4 l r-f-9 veled by the wagon during this time, consequently 9?/ 50 — %x-\- d _. =distance of the wagon from London,when 126 KEY TO ALGEBRA. 2a? met by B. But this distance is also =31-}- ~ therefore 50_2x+ 4 -^=31+| : . Again y+5 : 5 : : » : |, whence 3y-|-15=10a? and y= , substituting and reducing, we get the quadratic . 123a: 378 xktL x 2 = -s— . Whence a?=9, consequently y=25. 7. Given a? 2 -f-a?y=77, and xy — y 2 = 12, to find a? and y, by the Diophantine analysis. Ans. x—7. y=4:. 8. Three equal circles touch each other externally, and enclose between the points of contact a acres of ground, what are the radii of the circles ? A Ko,i6ik) i 9. A person has £21 65. in guineas and crown pieces, out of which he pays a debt of £14 17s., and finds that he has ex- actly as many guineas left as he has paid crowns away ; and as many crowns as he has paid away guineas ; how many of each had he at first ? Ans. 9 crowns paid away ; 12 guineas paid away. Suppose a?= the guineas paid away. And y=- the crowns paid away. Then 21a?-|-5#=297= amount paid out { . And 5a? +21^=249= amount on hand \ P er ) to find MISCELLANEOUS EXAMPLES. 127 ed after this reduction, but at the end of the 3d year he finds his stock exhausted; how much had he at the beginning; a(3n 2 +3n+l) Ans. . (n+1) 2 And a?*+y*=s275 ) x and V* Put Jipjx+bjyj^n. Then the first equation becomes n 2 .-- \-n=\0. Which equation gives n=5. Whence Jx-\-,Jy=5. From this last and the 2d equation, we find x=9, and t/=4. The following are from Bland's Problems, and involve equations only of the second degree. They are too severe for learners, but we are tempted to leave one or two of them, without solution, for the benefit of those who deem keys unnecessary. More of like character might be given. 18 7 x = . Ans. #=16 or 1. * Jx— 2 2x*(x*-\-cc i y=2x i (x-\-2a)-\-a 2 (x— a). Ans. x=\a or — a. *-f(«V— n+2*)* =7+2^- y* ; (3i/ — #+7 )* = ^. Ans. *=4, y=2, x—y Double the first equation, transpose — 2y 2 , and subtract 1 1 from both members, then we have 2y 2 +2x— ll+2(3i/ 2 — ll+2#)^=3+4y. Add y l to both members, and conceive the terms in the vin- culum to be P ; then P 2 +2P=3+4y+y«; Or, P+l=2+y. 128 KEY TO ALGEBRA. By restoring the value of P, and reducing, we have a?=6-f-2/ — y 2 . Put this in the 2d eq., &c. i 4. Given x 2 y — 4=4x*y — |?/ 3 , and x 2 — 3=#y ( x a — y* J, to find x and y. Ans. fl?=l, y=4. Put # =P, y a =Q, and we have P4Q2__ 4 = 4PQ2_|Q6 1 (!) P3_ 3 =PQ(P— Q), (2) Now put P=nQ, and eq. (1) becomes (4rt 4 -r-l)Q 6 — 16rcQ 3 =16. Conceive n to be a known quantity, then the last equa- tion is quadratic, and a solution gives 4(2tt 2 -r-2tt-r-l) _ 4 _ 4 4n 4 -p-l 2m 2 — 2/i-f-l 2n*— 2n-f2— 1* But from (2), Q 3 = 3 3 2 , =-— L-— Put the two values of Q 3 equal, and put n 2 — n-f-l=R, (3) Th6n 2^-I=i Whence 2 „= 6 |=? (4) But from (3), resolved as a quadratic, 2n=l±V4R^3 (5) From (4) and (5), 2R±2R^/4R~ 3~=6R— 3 ; Or ±2R74R— 3=4R— 3. Put V4R— 3=S, Then S 2 ±2RS=0, Or, S(S=b2R)=0. This last equation may be verified by taking either factor equal to zero ; and as the first factor only gives a rational quantity, we take that which gives R=f . By retracing, we easily find x and y. TECS BOOK IS DUE ON THE LAST DATE STAMPED BELOW AN INITIAL PINE OF 25 CENTS WILL BE ASSESSED FOR FAILURE TO RETURN THIS BOOK ON THE DATE DUE. THE PENALTY WILL INCREASE TO SO CENTS ON THE FOURTH DAY AND TO $I.OO ON THE SEVENTH DAY OVERDUE. EC 26 194f JUL 22 1942 AUG 5 W& FEB S 1947 LD 21-100m-7,'40 (6936s) \ 5-3 UNIVERSITY OF CALIFORNIA LIBRARY